You are on page 1of 127
Preface ‘This work blends together classic inequality results with brand new problems, some of which devised only a few days ago. What could be special about it when so ‘many inequality problem books have already been written? We strongly believe that even if the topic we plunge into is s0 general and popular our bookis very different, Of course, itis quite easy to say this, so we will give some supporting arguments. This book contains a large variety of problets involving inequalities, most of them difficult, questions that became famous in competitions because of their beauty and difficulty. And, even more importantly, throughout the text we employ our own solutions and propose a large number of new original problems. There are memorable problems in this book and memorable solutions as well. This is why this work will clearly appeal to students who are used to use Cauchy-Schwarz as a verb and want to further improve their algebraic skills and techniques. They will find here tough problems, new results, and even problems that could lead to research, ‘The student who is not as keen in this field will also be exposed to a wide variety of moderate and easy problems, ideas, techniques, and all the ingredients leading to a good preparation for mathematical contests. Some of the problems we chose to present are known, but we have included them here with new solutions which show the diversity of ideas pertaining to inequalities. Anyone will find here a challenge to prove his or her skills. If ‘we have not convinced you, then please take a look at the last problems and hopefully you will agree with ns. Finally, but not in the end, we would like to extend our deepest appreciation to the proposers of the problems featured in this book and to apologize for not giving all complete sources, even though we have given our best. Also, we would like to thank Marian Tetiva, Dung Tran Nam, Constantin Tanisescu, Ciilin Popa and ‘Valentin Vornicu for the beautiful problems they have given us and for the precious comments, to Cristian Babi, George Lasca and Callin Popa, for typesetting and for the many pertinent observations they have provided. ‘The authors Preface Chapter 1. Problems Chapter 2. Solutions Glossary Further reading Contents CHAPTER 1 Problems 8 Problems 1. Prove that the inequality Ves oP + VES + holds for arbitrary real numbers a,b,c Kémal 2. [Dinu Gerbinescu ] If a,b, € (0, 1) prove that Vabe + (Gala <1 Junior TST 2002, Romania 3. [ Mircea Lascu ] Let a,0,¢ be positive real numbers suc that abe = that Prove b Pai ee Oe > Vat Vb+ e438. Wat Te oe 2 NaH Mbt ve Gazeta Matematica 4. If the equation x* + ax? e+ bet Ohas at least one real root, then +P D8, ‘Tournament of the Towns, 1993 5. Find the maximum value of the expression a9 + y* +2" —3ry2 where «? + y? + 2?= Land a, y,2 are real numbers 6. Let a.b,c,2,y,2 be positive real numbers such that = 1. Prove that 2JGyt wet aNab+ bet ca) Dav Dats bet bV WEF ca + VIE + ab, Gazeta Matematicd 9. If a,b,c are positive real numbers such that abe = 2, then a +A >avb Fer byeta+evath. (Old andl New Ineawalities ° ‘When does equality hold? JBMO 2002 Shortlist, 10. [Toan Tomescu ] Let ay, 2 > 0. Prove that aye (1+ 32)(@ + 8y)(y + 92)(2 + 6) When do we have equality? Gazeta Matematica 11. [ Mihai Piticari, Dan Popescu ] Prove that 5(a? +H? +c?) < 6a +H +8) 41, for alla,b,c>Owitha+b+e= 12. [ Mircea Laseu ] Let 21,25... € Ryn 2 2and a > Osuch that ay + 2a motetmy=aand aitaht.. 42 < 2 Prove that 21 € p 46 {1,2,...,n} |: for all 18. [ Adrian Zahariue | Prove that for any a,},¢ € (1,2) the following inequality holds ve ev evo __ ave Bye-ola* toja— avi dav be 14. For positive real numbers a,b, ¢ such that abe < 1, prove that $2bsloatbte 15. [ Vasile Cirtoaje, Mircea Lascu ] Let a,b,¢,,y,2 be positive real numbers such that a+2> b+y>e+2and atb+e=a+y+z, Prove thatay+br> acts:. 16. [ Vasile Cirtoaje, Mircea Lascu ] Let a,b, ¢ be positive real numbers so that ibe = 1. Prove that abe = 1. Prove that 4 5 + anb4e 2 abeacthe Junior TST 2003, Romania 17. Let a,b,c be positive real numbers. Prove that @ 8 Ole Be Bra ab eta JBMO 2002 Shortlist 10 Problems 18. Prove that if n> 3 and x),22,...,q >0 have product 1, then 1 1 1 4 +t Tay tang Fay rem Tq Fino Russia, 2004 19. [ Marian Tetiva | Let 2,y, z be positive real numbers satis! the condition wey? + 22+ 2rye=1. Prove that a) ayz< 20. [ Marius Olteanu | Let x) ,22,20g,24,25 € R so that xy +2 +23+24+25=0. Prove that |cos 1] + [e082] + |e0s.79| +| cos:r4|+ | costs] > 1 Gazeta Matematica 21. [ Florina Carlan, Marian Tetiva | Prove that if x,y, 2 > 0 satisfy the condition atytz=aye then xy +22 + yz 23+ Vet+1+ YF + VEFT 22, [ Laurentiu Panaitopo! | Prove that for any real numbers 2, y,2> JBMO, 2003 23. Let a,b,c> Owith a+b-+¢=1. Show that @ab Bee eta > bee tera abe 24. Let a,b,c>O such that at + b+ of <2(0°H? + We? + ea), Prove that a +R +e <2abt bet ca), Kvant, 1988 (Old and New Ineavalities 1 25, Let n> 2and 2y,...,q be positive real numbers satisfying, Prove that Vietnam, 1998 26, [ Marian Tetiva | Consider positive real numbers 2, y, 280 that Prove the following inequalities a) ay 227; b)ay +22 + ye > 27, e)atyt d)aytre+ DActyt2+9. 27. Let 2, y,2 be positivereal numbers with sum 3, Prove that VE+ U4 Vib ay tyr ton Russia, 2002 28. [D. Olteanu ] Let a,b,c be positive real numbers. Prove that ath a bee bcta 8 b+ ~ rash Gazeta Matematica beera ard arbte cba 29. For any positive real numbers a,b,¢ show that the following inequality holds ab eceta ath bre Bet a-esb ave bea India, 2002 30. Let a,b, be positive real numbers. Prove that Blab + be + ea) tere Baacta Foab+h~ atb+e Proposed for the Balkan Mathematical Olympiad 31. [ Adrian Zahariuc ] Consider the pairwise distinct integers 17,2, n=0.Prove that hbo} tet ak D> aura oate tt ancy On 2 Problems 82, Murray Klamkin | Find the maximum value of the expression 2.72 + a3.t5 + seb a2 yg they When 24,29,.--.n-1tn 2 O add up to Land n > 2 Crux Mathematicorum 33. Find the maximum value of the constant ¢ such that for any 1, 2,0++.Tqy' 1+ > O for which 2p 4) > 1 + ra +++ + zy for any k, the inequality Vii+ Veto + Yin Sova Fee also holds for any n. IMO Shortlist, 1986 34. Given are positive real numbers a,b,¢ and 2, y,2,for which a +2= c+ 2=1.Prove that totia (obesayey(he had Russia, 2002 35. [ Viorel Vajaitu, Alexandru Zaharescu ] Let a,b,c be positive real numbers, Prove that ab be ca 1 abe R Teena oaaam 2. Prove that aya}+ aya! ++ anal > axa! + aah + a+ aval, Tran, 1999 39, [ Mircea Lascu ] Let a, bye be positive real mubers. Prove that ath (a be ato, 21+), e 2a(S+4+5) bte cta (Old andl New Ineawalities a 40. Let a1,42,.--s0n > 1 be positive integers. Prove that at least one of the numbers 9, 5... °-yOn, V7 is less than or equal to 3. Adapted after a well-known problem 1. [ Mircea Lascu, Marian Tetiva ] Let 2,y,2 be positive real numbers whieh satisfy the condition ay tae tye Qrye Prove that the following inequalities hold a) yz S53 byrtyte2: o) bate bod@tyss) 1.1.1 (22-17 . Ret ae bye) > EA ps where = ma 42. [ Manlio Marangelli | Prove that for any positive real numbers x,y, 2, Baty + ye + 0\(0y? + ye? + 20°) Bayete ty +2) 43, [ Gabriel Dospinescu ] Prove that if a,0,¢ are real numbers such that max{a,b,¢}~min{a,d,c} <1, then +P ++ Gabe > 30) + B+ 3a 44. | Gabriel Dospinescu | Prove that for any positive real numbers a,b,c we have a ee & 1.1 = (22) (2-2) (2+) ouusvea(Lebed). a n+ Prove that 1b cay <1 TST Singapore 46. [ Clin Popa ] Let a,b,c be positive real mumbers, with a,b,c € (0,1) such that ab + be + ca= 1. Prove that a( 47. [ Titu Andreescu, Gabriel Dospinescu ] Let ,y,2 < land 2+ y+2=1 Prove that BP 1-e Pt ). a 7 ul Problems 48. [ Gabriel Dospinescu ] Prove that if YF + V+ V2= 1, then (1-2)? y= 2)? > 2 ayer + u)ly + 2)2 +2) 49. Let 2, 4,2 be positive real numbers such that yz = «-+y+2+2. Prove that (1) ayt+y2422>Acty+2); ©) VE+ Vi+ VES SVE 50. Prove that if «,y, are real numbers such that x? + y® + 2? =2, then atytesaye +2 IMO Shortlist, 1987 BL. [ Titu Andreescu, Gabriel Dospinescu ] Prove that for amy 21,.02).-.431_ € (0,1) and for any permutation o of the set {1,2,...,m}, we have the inequality ae 52, Let a1,.22,..-5ty be positive real numbers such that }> 7 = 1. Prove that » » Lvs -IY Vojtech Jarnik 53. [ Titu Andreeseu ] Let n > 3 and a),42,...,@n be real numbers such that aybaat..+dy > n and af-+a8-+...-a2 >n?. Prove that mar{ay,4,...éy} > 2 USAMO, 1999 54, [ Vasile Cirtoaje] If a,b,¢,d are positive real numbers, then a-b bre e~d d-a bec etd dva* athe” 55. If x and y are positive real numbers, show that 2¥ + y* > 1 France, 1996 (Old andl New Ineawalities Fry 56. Prove that if a,b,c >O have product 1, then (a+ b)(b+e)(c+a) > 4(a+b+e-1). MOSP, 2001 57. Prove that for any a,d,¢ >0, (+8 +2 )a+b—o)(b+e—a(c+a—b) < abc(ab+ be+ ca) 58. [ D.P.Mavlo ] Let a,,¢> 0. Provethat b (a+I(O+ Nle+1) Beatbtcrtttatg Sy « ab a8 1+abe +f23 Kvant, 1988 50. [ Gabriel Dospinesen ] Prove that for any positive real numbers 21,.225.-+49 with product 1 we have the inequality 60. Let a, b,c, > O such that a ++ ¢= 1. Prove that a 34 8+ minf1 1.4 a +B +8 + abed > {igte Kvant, 1993 61. Prove that for any real numbers a,b,c we have the inequality Vata Ps PP a= obo > (1+ aL FPL + A Ma b)%(b- 0) (ea)? AMM 62. [ Titu Andreeseu, Mircea Lascu | Let a,2, , 2 be positive real numbers such that ayz=1 and a > 1. Prove that a yt at pte 24a 3ty 3 =2 63, Prove that for any real numbers 21,...,Pn,iy+++,n Stich that af +++ = vite tyah San). é (ery ~ rm)? <2 ( Korea, 2001 16 Problems 64. [ Laurentiu Panaitopol ] Let a1, @2,...,n be pairwise distinct positive inte- gers. Prove that 1 2n pate 4 at> Ma bay tet a9) TST Romania 65. [ Calin Popa ] Let a,b,c be positive real mumbers such that a+b +e = 1 Prove that te ead 3v3 al Ve + Vab) ” 030+ Voc) © o(V3b+ Yea) ~ 4 66. Tit Andeescu, Gabriel Dospinescu | Let a, 6, c,d be real numbers such that (140%) +8) +2)(14 #) =16. Prove that -3 9{ab + be + ea) for any positive real numbers a,b, ¢. APMO, 2004 68. [ Vasile Cirtoaje | Prove that if0 abe Prove that at least two of the inequalities 218 6562,3,6.52,8, 6.5, aE eae aT are true, TST 2001, USA 70. [ Gabriel Dospinescu, Marian Tetiva | Let 2, y,z>0 such that ntytraonye Prove that (x —1)(y -1)(z -1) <6v3-10. (Old andl New Ineawalities ra TL. [ Marian Tetiva | Prove that for any positive real numbers @,0,¢, bP +(b- oP +(e~ a)? 7 Moldova TST, 2004 ‘72. [ Titu Andreeseu ] Let a,b,c be positive real numbers. Prove that a? +3)(H— #43) +3) > (a+ b+0)%, USAMO, 2004 73. [ Gabriel Dospinescu ] Let n> 2 and 2),22,...,tq > Osuch that Prove that 74. [ Gabriel Dospinescu, Mircea Lascu, Marian ‘Tetiva | Prove that for any po- sitive real numbers a,,¢, PLR ees abe + 3D (1+ all +149) 75. [ Titu Andreeseu, Zuming Feng ] Let a,b,c be positive real numbers, Prove that Qatb+e)? | Qbtare , c+a+o) 2a + (5+ oF (ato 26 USAMO, 2003 76. Prove that for any positive real numbers x, y and any positive integers m,n, (n—1)(m—1)(0™" $y") (m-tm—1)(2y"42%y) > mally ey") Austrian-Polish Competition, 1995 77. Let a,b,c, d,e be positive real numbers such that abede= 1. Prove that sabe b+ bed ctade | d+dea creab_. 10 ab+abed” Tbe bode” 1 od + clea” 1+ de +deah” T+ ea+ cabo = 3” Crux Mathematicorum 18 Problems '78.[ Titu Andreescu ] Prove that for any a,b,c, € (0 3) the following inequality holds sin(b sind n(b— 0) -sin( ime -sin(c~ a) -sin(c ~ 6) sin(e+ a) sin(a +0) ‘TST 2003, USA 20. 79. Prove that if a,b,c are positive real numbers then Vr + VPP a PAF Oa > Varb+ Het a+ Val Fb + at KMO Summer Program Test, 2001 80. [ Gabriel Dospinescu, Mircea Lascu ] For a given n > 2 find the smallest constant ky, with the property: if a,,....a,> 0 have product 1, then Witenes ay) * pay) + ap) 81. Vasile holds toaje | For any real numbers @,b,¢,,y,2 prove that the inequality 52 art by +24 VTP +e PHS Flat b+ ele tut Kvant, 1989 82, [ Vasile Cirtoaje | Prove that the sides a, 0, cof a triangle satisfy the inequality a bie a(gab=£-1) 83. [ Walther Janous ] Let n> 2 and let 21,22, > Oadd up to 1. Prove that » » 1 non +1+)> M(+z)= MCE) Crux Mathematicorum 84. [ Vasile Cirtoaje, Gheorghe Eckstein ] Consider positive real numbers 1 Tay ony dy SUCH that 3 2..t%y = 1. Prove that 1 1 L tuto <1 Tem a-l+m a-Tti TST 1999, Romania 85. [ Titu Andreesen ] Prove that for any nonnegative real numbers a,b,¢ such that a? + 68 40? + abe =4 we have 0 < ab + be + ca — abe <2. USAMO, 2001 Old and New Ineaualites 10 86. Titu Andreescu ] Prove that for any positive real numbers a,b, the following, inequality holds atbte 3 — Vabe < max{( a — vb)*, (vb — ve), (ve- va)”} ‘TST 2000, USA 87. [ Kiran Kedlaya | Let a,b, be positive real numbers, Prove that a+ Vab+Vabe . of att arbre aes 88. Find the greatest constant k such that for any positive integer n which is not a square, |(1+ yn) sin(ryn)| >. Vietnamese IMO Training Camp, 1995 89. [Dung Tran Nam ] Let 2,y,2 > 0 such that (2 + y +2)" = 92rys, Find the minimum and maximum of ma Vietnam, 2004 90. [ George Tsintifas ] Prove that for any a,b,c,d >0, (a+ G+ c+ d)(d +a)" > 6a AP(a+b+e+ Ad Crux Mathematicorum 91. [ Titu Andreescu, Gabriel Dospinescu ] Find the maximum value of the ex- pression (ab) (bed (ea) T=ab Tokina where a,b, ¢ are nonnegative real numbers which add up to 1 and n is some positive integer, 92. Let a,b, ¢ be positive real numbers. Prove that to tt a(i+8) Kite eli ta) ~ Yabo + Yabo) 93. [ Dung ‘Tran Nam | Prove that for any real numbers a,b,¢ such that a2 +5?-+ 9% 2a+b+0) —abe <10. Vietnam, 2002 20 Problems 94. [ Vasile Cirtoaje] Let a,b, ¢ be positive real numbers, Prove that (+$-1)(s-4-2)(oo4-a)(cadrJe(co!-a) (arts) oe 95. [ Gabriel Dospinescu ] Let n be an integer greater than 2. Find the greatest real number m,, and the least real umber M, such that for any positive real numbers Ty Maye Ty (With ye nm), mn Dean Data Tota 0 prove that 2(a3 + (H+ Ile + 1 +1) > (1+ abed\(1+ a7) +H )U+e)0+ 2) Gazeta Matematici 98, Prove that for any real muanbers a,b,c, (a+ +b+oft(c+a)t> ala 4 bi +e), Vietnam TST, 1996 99, Prove that if a,b,c are positive real numbers such that abe = 1, then 1 1 1 1,1 4 Travh Trove Theva S240" deb Tre Bulgaria, 1997 100. [ Dung Tran Nam ] Find the minimum value of the expression + + ; + 2 where a, b,¢ are positive real numbers such that 21ab + 2be + Sea < 12, Vietnam, 2001 101. [ Titu Andreescu, Gabriel Dospineseu ] Prove that for any’, y,2,0,0,¢>0 such that ay +yz+27=3, b c pat Dt gett yetw es (Old and New Ineavalities a 102. Let a,),¢ be positive real numbers. Prove that (b+e-a)? | (C+a- | (atb—c)? CF ra (eral Te aries Japan, 1997 108. [ Vasile Cittoaje, Gabriel Dospinescu ] Prove that if a ,a2,...,ay > 0 then My tag tor tan. ) a Toe af tal t+ ah — naan. an >(n-1)( where ay is the least among the numbers 4; 42... 104. [ Turkevici ] Prove that for all positive real numbers x,y, 2,t, att Abts deytrey tye tee tea tart ryt, Kvant 105. Prove that for any real numbers a1,a2,...,dy the following inequality holds (5 «) < > wea im 106. Prove that if a1,4495...,@n)D14+.5bn are real numbers between 1001 and 2002, inclusively, such that aj +a} +++++ a% = 6} + bj ++++4 6%, then we have the inequality th be b, =H Gn) TST Singapore 107. [ Titu Andreescu, Gabriel Dospinescu ] Prove that if a,,¢ are positive real numbers which add up to 1, then (PHPYVFVE +a) > + PE + ea)? 108. [ Vasile Cirtoaje ] If a,b,c, dare positive real numbers such that abed then 1 1 1 1 Trapt Oso ee ae Gazeta Matematica 109. [ Vasile Cirtoaje ] Let a,b,¢ be positive real numbers. Prove that a ee é a b e BLO” Bed aR Ape ota ath Gazeta Matematica 2 Problems 110. [ Gabriel Dospinescu ] Let a1,42,.-.s4n be real numbers and let S be a non-empty subset of {1,2,...,n}. Prove that (=e) < TY tetas ie acigjen TST 2004, Romania 111. [Dung Tran Nam J Let 21,9... 22001 be real numbers in the interval [—1, 1] such that 2?+23+...+a%jgq=0. Find the maximal value of the 2) +2 +-+-+ 22008 112. [ Gabriel Dospinescu, Clin Popa ] Prove that ifn > 2 and a1,02,.--50n arereal numbers with product 1, then 2n abet tak=n> 2 Vin Tay + a2 tos + On — 113. [ Vasile Cirtoaje ] If a,b, are positive real numbers, then NEVE s ora 114, Prove the following inequality for positive real numbers 2, y,2 rot, tye ortnsea( et gape) a Gazeta Matematics Tran, 1996 115, Prove that for any x,y in the interval [0, 1], Vises Vier + VO=2P + oP 2 (1+ vi)(1— ay) 116. [ Suranyi ] Prove that for any positive real numbers a;,a2,...,dn the fol- lowing inequality holds (n=Vfap af +---bah)-+ nara. ...dn > (ayHaate + ay)(af bay". bann!) ‘Miklos Schweitzer Competition 117. Prove that for any 2,2,-..,tq > Owith product 1, DY wi-ayt2d8 icigicn 1. ‘A generalization of Turkevici’s inequality (Old andl New Ineawalities 2 118. [ Gabriel Dospinescu ] Find the minimum value of the expression ae m= 1a; where ar, a2, add up to 1 and n > 2is an integer. real numbers such 119. [ Vasile Cirtoaje ] Let ay, a2,.-. ay <1 be nonnegati that [aR Ea , VB ee) Prove that a wate 120. [ Vasile Cirtoaje, Mircea Laseu ] Let a,b,c, 2,1, 2 be positive real numbers such that (a+b+oetyt2 al +P + 2)(2" ty? +2 Prove that besye 2, find the minimal value of the constant kp, such that if 21,29,...,2n > 0 have product 1, then 1 1 1 ae set eee Vit kaa, VIF Eat Vit Rata ‘Mathlinks Contest 122, [ Vasile Cirtoaje, Gabriel Dospinescu ] For a given n > 2, find the maximal value of theconstant ky such that for any 27 ,:12,...,dtq > 0 for which x? +234 --+ 3.=1 we have the inequality (1=ai)(1 = a2)... ty) 2 Rnitata «tn. CHAPTER 2 Solutions 20 Solutions 1. Prove that the inequality TTT + VERA + holds for arbitrary real numbers a,b,c 3v2 (Ia Kémal First solution: Applying Minkowsky’s Inequality to the left-hand side we have Vea IAF PSP VES Denoting a +b+ c= we get (a+b and the conclusion follows. Second solution: We have the inequalities VERE + FEOF + VES OFS 5 l=, Weld [de “Ve v2 3y3 [La] 1 for allreal muaubers x the last quantity is atleast “5 and because 2. [Dinu §erbiinescu ] If a,b,c € (0, 1) prove that Vabe+ (T= al I} <1 Junior TST 2002, Romania Observe that a4 <4 for x € (0,1). Thus Vato < Yate and (T= W-l 9 < fa —-— o}. By the AM-GM Inequality, Vabe < Vabe < A - and Tata 9 < Yaa ag < UHM TU Helo) (Old and New Ineavalities a Summing up, we obtain Vabe + V=ayl- p< gabeestoesttits as desired. Second solution: ‘We have Vabe + /1= a) (t= HI) < vb. e+ VI-B-VI=e<1 by the Cauchy-Schwarz Inequality. Third solution: Let a = sin? x,b becomes: Jin? yc = sin? z, where 2,y,2 € (0, a ‘The inequality sing siny-sin2 + cose -cosy-cos2 <1 and it follows from the inequalities sing siny-sinz + cosx- cosy -cosz < sin siny + cosz- cosy = cos(z —y) <1 3. [ Mircea Lascu ] Let a,d, be positive real numbers such that abe = 1. Prove that > ee SE ovat vO4 ve+3. Gazeta Matematies = ( es (2) +(/F+ 2).(\2 vee 2 Va+ VE Va) > Vat Vo+ Ve Vee = Vas Vo+ e+. Solution: Vv Qe? tbe $1 4. If the equation 2‘ +a LP D8. O has at least one real root, then ‘Tournament of the Towns, 1993 2 Solutions Solution: Let x be the real root of the equation. Using the Cauchy-Schwarz Inequality we infer that BS rae because the last inequality is equivalent to (x? =1)* >0. 5. Find the maximum value of the expression 2? + y+ 2% ~3zy2 where 2? +y? + 2 = Land x, y,2.are real numbers, Solution: Lot t= ay + yz + 2x. Let usobserve that (2 + p+ 28 —Bxye)? Pl — ay — ye — 20)? = (1+ 201 4) ety We also have +4 0. Prove that Ver OPT + ETRE A+ VET OPT Sav at The + bV WEF ca + eV IEF ab, Gazeta Matematica Solution: We start from (a?—62)? > 0. We rewrite it as dat +400? 4404 It follows that Yat + a0? +B? > v3 (a? + 6°) Using this observation, we find that (XVeFFFTR) > 3(De) But using the Cauchy-Schwarz Inequality we obtain (Covert) < (Le) (Lee +9) avb tet byerat eva +b. When does equality hold? JBMO 2002 Shortlist First solution: Applying the Cauchy-Schwarz Inequality gives BP? +P +e) >3atb+e" (A) and (+P LAP <(atb+Q@+h+A), (Q) ‘These two inequalities combined yield Pavia » CHBL\+b+9 = 3 24+ e+ +(at+e)+(a+b)] ViFe+ WaFEbeVETD! a) —————eorv 8) Using the AM-GM Inequality we obtain —_ avbte+byatetevash > B4fabe( (aFWb+ Nera) 2 3i/abeVBabe =3V8= (aVore+ Yates eVard) 2O(avire4 bVare+evard). (4) ‘The desired inequality follows from (3) and (4) Second solution: We have avb+e+bVetatevath< Verte Using Chebyshev Inequality, we infer that VUETE Mar b+d < VOTE FA) rueby the AM-GM. fats 6 and so it is enough to prove that a® + 6? +c? > 3abe, whieh is Inequality. We have equality if a=b =e = V2. 10. [Ioan Tomeseu] Let 2,2 >0. Prove that aye Bayle + yy + 92) a (Old andl New Ineawalities a When do we have equality? Gazeta Matematies Solution: First, we write the inequality in the following form (isan (12%) (1-2)(1-2) 2° But this follows immediately from Huygens Inequality. We have equality for ra2y= 11. [ Mihai Piticari, Dan Popescu ] Prove that 5a? +H +c) < 6a +H +8) 41, for alla,b,c>Owitha+b+e=1. Solution: Because a+ b-+0= 1,we have a® + 684.0! =3abe + a?-+ b? 4+ 0? — ab — be — ea. ‘The inequality becomes SQ? + 8 4e?) < 18abe+ Gla? +b? +c?) — Gab + be + ca) +1 ISabe-+1— 2(ab-+ be-+ ea) +1 > O(ab + be + ea) 8(ab + be + ea) <2 + 18abe 4 (ab-+ be + ca) <1 + 9abe (1 2a)(1— 26)(1— 2c) < abe 4 & (b+ e-alle+a—B(a+b—0) O uel that 21 + fyb eot ty =a and e] ta} +... 42% < 2. Prove that 2; € p 22) for a 4€ {1,2,...,n} Solution: Using the Cauchy-Schwarz Inequality, we get (a= ny)? <(n-1)(2} +28 + a) Abvac, the last one coming from a +b > 2Vab and b +c > 2Vbe. Writing the other two inequalities and adding them up give the desired result. 14. For positive real numbers a,),¢ such that abe < 1, prove that oie oo atbee First solution: Tfab +be+ea 3pand +4 > Be. Adding these three inequalities, the conclsion is immediate Forth solution: ofa _ ofea® | _ oft Let r= ff S.y= D2 5. Consequently, a= xy’, = ye, and also ryz < 1. Thus, using the Rearrangement Inequality, we find that a_ 2 . ot 3 _ De-LE ow LE=LeeLa=Le 15. [ Vasile Cirtoaje, Mircea Laseu ] Let a,b,¢,2,y,2 be positive real numbers such thata+2>b+y>c+2andatbte=a+y+z, Prove thatay +br> acta2, =n, Solution: Wehave aly-o)+x(0— (a+b—2-2)-+2(b-2) = a(a-2)+(a+2)(b—2) = 1 He? 2) + (atay(b—2) = Ha- a)? + Hata)la—2+2)-22)= 2 1 (a- 2) + 3(a + a)b-et+y—2)20. ‘The equality occurs when a= 2,b = 2,¢ =yand 2x>y +2. 16. [ Vasile Cirtoaje, Mircea Lascu ] Let a, ,¢ be positive real numbers so that abe = 1. Prove that 5 5 Ty bre? wrath Junior TST 2003, Romania Solution: We set 2 = equivalent to a Solutions From (x+y +2)? > 3(ry + yz + 22) we get 14—+—_ > 1+ —"_. ay syste ty tae 0 it suffices to prove that 9 6 14% 5 _ 6 * Grytel ates ‘The last inequality is equivalent to ( 1- sn) > O and this ends the proof, 17. Let a,b,c be positive real numbers, Prove that a le Be pirate gtett JBMO 2002 Shortlist First solution: We have aa Bz Gran bea + HD abla +0) &(a—B%(a+0) 20, which is clearly true, Writing the analogous inequalities and adding them up gives @ a ee 2 a2 Pe Brat Ge Gta bt Sabet See aa Fa Second solution: By the Cauchy-Schwarz Inequality we have oe frosa( otha 0 we only have to prove that a Boe Es Spathre But this follows immediately from the Cauchy-Schwarz Inequality. 18. Prove that ifm > 3 and x),22,...,q >0 have product 1, then 1 1 1 + > 1 Tq Fino Tay tang Fay rem Russia, 2004 Solution: We use a similar form of the classical substitution Ti this case the inequality becomes a a fn me Gta ta aba ty atte (Old andl New Ineawalities 2 and it is clear, because n> Sand a;+ ae + ai42 < a1 + a2 +++ + dy for alli. 19. [ Marian Tetiva | Let 2,y, z be positive real numbers satisfying the condition wey? + 22+ 2rye=1. Prove that a) ayz S53 byrtutesh: Omtartps tc eytee 4) ay +224 yes 5+ Daye Solution: a) Thisis very simple, From the AM-GM Inequality, we have 1 1 Bey be rye d WPT aly < 5 ayes 5. 1b) Clearly, we must have x,y,2 € (0,1). Ifwe put s=2+y-+2, we get immediately from the given relation 8 -2941=2(1-2)(1-g)(1-2) ‘Then, again by the AM-GM Inequality (1-2, 1 y, 1— 2 being positive), we obtain 4 a net go(Iebtou tt ) =2(53) After some easy calculations this yields, 2s) 49s? — 27 < 04 (28-3) (s+.3)° <0 and the conelusion is plain. ) These inequalities are simple consequences of a) and b): (ruts? 3 tyes 3 ry tsztye + 5 and 13 wayte n 1.3 Pay + Daye Z1-25=5 d) This is more delicate; we first notice that there are always two of the three 1 numbers, both greater (or both less) than 5. Because of symmetry, we may assume 1 1 that 2. < 5, ora.y> 5 and then (22-1) (2y-1)20¢r+y-2ny < 26 Solutions On the other hand, 1 apy + 2? + drys > dry +2 + Qey2> => Qey (142) < 1-27 ry < 1-2. Now, we only have to multiply side by side the inequalities from above aty-am0, because t and ; are both greater than 1 Remark. 1) One can obtain some other inequalities, using e+2ryS1 and the two likes y+ ez 0 verify a=sin Pays 2 Daye then there is a triangle ABC’so that According to this, new proofs can be given for such inequalities, On and New Ieuaies a 20. [ Marius Olteanu | Let x) 272,03, 4,25 € R so that ry +r +g +24 +25=0. Prove that Jeoszi| + |cosx2|+ | cos 23| + | cosirs| + |cos.xs| > 1. Gazeta Matematica Solution: It is immediate to prove that |sin(x + y)| 0 satisfy the condition atytzeaye then zy + vz +y2 234 Va7+1+ Vy tlt veFt1. First solution: We have ayz=atyt2>2yayt25 (Va) —2Vay- 220. Because the positive root of the trinomial 2t?— 2t — 2 is livig? wwe get from here 1+ VIFF yap2 EES 3g symz is vise Of course, we have two other similar inequalities. Then, aytazty: > afpetwrtevm> > 34 VF FT + Vet + VEST, and we have both a proof of the given inequality, and a little improvement of it, 28 Solutions Second solution: Another improvement is as follows. Start from 1 11 +e Fetes tet eisey riveree, a a2 ginty y yf v which is covalent to (ay +22 + yz) b days (ety tate? =3(etyt2) Further on, (ay + 2 + yo — 3)" = (ay + 22 + yz)" — 6 (cy + ez + y2)+9> DB (ety +2) —6 (ey tae + yz) +9=3(22 +9? +22) 49, ‘so that. ay +024 ye23+ VIET ETS But VIG P Ee o2 VPs + VPs VFI is @ consequence of the Cauchy-Schwarz Inequality and we have a second improve- ‘ment and proof for the desired inequality aytaztyz > 3+ /3@ FHT > 84 VeF1+ VF 1+ VeF1 22, [ Laurentiu Panaitopo! | Prove that for any real numbers x,y, 2 > JBMO, 2003 Solution: Let us observe that 9 < and the similar relations. Setting a= 1+ 2?,b=1+ y?,¢= 14 2° it is sufficient to prove that ob ye Deeb Sate Bra (Old and New Ineavalities 209 for any a,,¢>0.Let A po AdiC= 28 |B 9 e+ b, B= 2a+¢,C=% +a. Thena 4A— and the inequality (1) is rewritten as A +4)e15 a2 and > Sand the conclusion follows. ‘Ah allémafive solution for (1) is by using the Cauchy-Schwarz Inequality: a b c @ 2 2 ( ? 4+p% = > 2e+b 2ate W2+a 2ac+ab 2ab+ch 2%8e+ac~ 3(ab+ be + ca) at 28, Let a,b,c> Owith a+b+e=1. Show that 2 Pte eta + > bee ora abo Solution: Using the Cauchy-Schwarz Inequality, we find that by (Seta) bre Fae +q + na oad And so it is enough to prove that ya 1+(L#) 22 e+q+2E at. ‘The last inequality can be transformed as follows 14(Se) > 2Veosg+2 abo 1+(Le!)'> > Le2Le2Lw + (Le) and it is true, because Lez ye (Chebyshev's Inequality) and (rey 40 Solutions 24. Let a,b, ¢> O such that at + bf + of <2(076? + Pe + ca”), Prove that a +R +e <2abt bet ca), Kvant, 1988 Solution: ‘The condition Yarrow is equivalent to (a+b+e)la+b—c(b+e—al(e+a—b) 20. Inany of the cases a= teb= Veis2yab is clear. So, suppose a+b # c, b-+e #a, c+a¥ b. Because at most one of the numbers b +¢—a, ¢+a—b, a+b —c is negative and their product is nonnegative, all of them are positive. Thus, we may assume that +5, the inequality +a, c= a 2and 2),.., 2, be positive real numbers satisfying 1 1 1 1 FS * FTO FF TONS ~ TOOK Prove that nam, 1998 Solution: Let aig = ai. The problem reduces to proving that for any positive real rutabers dy, @25:- yn such that a1 + ay-++++-+ ay = 1 we have the inequality )) > a= ‘This inequality can be obtained by multiplying the inequalities 1 ation tate ston a ate Vv (n=1) Old and New Ineaualites a 26. [ Marian Tetiva | Consider positive real numbers 2, y, 2 0 that eet tomy, Prove the following inequalities a) ayz > 27; b)ay +224 y2> 27; tty t229; A ay toe tye @Wetyr2+9. Solution: a), b), ¢) Using well-known inequalities, we have ays sat ty? $2? BYP ye S (ayz)® > 27 (aye) which yields ry2 > 27. Then ay tart yz > 34) (aye) 2 VIF = 27 and atyt2>3yaye> BVI =9, 4) We notice that 2? <2y2 a 1 Hence all the three numbers must be greater than 1. Set a=r-1,b=y-l, We then have a>0,5>0,e>Oand r=atly=b+l, Replacing these in the given condition we get (a+ 1% +(b 41" +(e +17 =(a+1)(b+1)(c+1) which is the same as aR LE+atb+e+2=abe + ab + act be If we put g=ab + ac-+ be, we have q 12. Now, recall that a = 2 -1,b=y—1,¢=2—1, sowe get @-)W-) + @-(e-)+W-NE-)22S S mytartyz22(etyt2t9 and we are done, One can also prove the stronger inequality aytartyz24(ety+2)-9 Thy it! 27. Let 2,4, be positivereal numbers with sum 3, Prove that VE + i+ Vez ay tye ten, Russia, 2002 Solution: Rewrite the inequality in the form wt aety + Qyyt rt Wer a2 ty? + 274 Dry t dye tere eaves yl +2742? 4 2v229. Now, from the AM-GM Inequality, we have a4 2vEaa"4 V4 Vez aera = 32, y+ 2yy> By, 27+ 2V2 > 32, hence Pays t42(vE+ Vi+ve) > Het y+2)29. 28. [ D. Olteanu ] Let a,b,c be positive real numbers, Prove that a+b a bee, _b i cta_e¢ 3 bye Wtbee cha Beetva ath wrath 4 Gazeta Matematies (Old and New Ineavalities 4 Solution: Weset r=a+b,1 the equivalent form becand: a+ candafter a few computations we obtain z eta u y ye But using the Canchy-Schwarz Inequality, yoy ey tye) yl 2tr ety yee phe atte ot 7 ery — > ——_Setueet Dey + yet allay + ve +e) = yt ye tert (ety +2) and the conclusion follows. 29. For any positive real numbers a,b,¢ show that the following inequality holds India, 2002 2. Observe that ate bre 1-2 T¥0 Using similar relations, the problem reduces to proving that if 2yz= 1, then e-l y-1 en retest esr20* © (2 =1)(2 41) Fy? -(etF(?=1lyt 20 & @ Yet Yate Pres But this inequality is very easy. Indeed, using the AM-GM Inequality we have Yl x*z > 3 and so it remains to prove that }> x? > 7, which follows from the inequalities sels, 30. Let a,b,c be positive real numbers. Prove that > abe = Proposed for the Balkan Mathematical Olympiad “a Solutions First solution: 3{ab + be-+ ca) Sineea+b+e>* it suffices to prove that: a+b @ 8 é Botte cata Page zt tite From the Cauchy-Schwartz Inequality, we get e for Die 2 Ease) ‘Thus we have to show that (P+P +E) > (atb+e)-Y al be+ 2) ‘This inequality is equivalent to a4 bts abela +O +0) > abla? +B?) + belb? +2) + eae? +0) which is just Schur’s Inequality, Remark. ‘The inequality s2athte a ® é Pater Focatd Poabeb was proposed by Vasile Cartoaje in Gazeta Matematica as a special case (n = 3) of the more general inequality 2a" — 6" Second solution: Rewrite our inequality as b+ a® Bao = But this follows from a more general result: Ha,bje,2,y,2>0 then But this inequality is an immediate (and weaker) consequence of the result from problem 101 Third solution: Let ‘Via (Old and New Ineavalities 45 and & b+ 0)(¥ — be +c Bee ee So we get see = (De)(Cees pero -te+d) (Carers) aL tality. Hence 2a= DL vats vive- Soa from the Cauchy-Schwarz Ine« 423(L4m) We denote os ee Savrara and the similar relations with As and A.. $0 A> Au-+di+de. But because (Sa) > 3(T a0) we get eC | oy, ; (EX ore IVF and also the similar inequalities are true. So we only need to prove that x ey sJpecteone We consider the convex function f(#) wwe finally deduce that Ee Gs) 2 We have equality if and only if a= =. 31. [ Adrian Zahariuc ] Consider the pairwise distinct integers 27 ,2%2,...,n n=0.Prove that ait ap tee + a2 > ayty t worg tert aati $ Qn 3. 46 Solutions Solution: ‘The inequality can be rewritten as i= 21)? 2200-3) mi Let my =min{27y,22,...,7,} and ayy = max{ri,2,. of generality that m < M. Let 9}, Suppose without loss $1 = (Gm = tm)? oe + (et = wae)? and Sa = (eM — rags) t+ na 21)? + (1 — 22) +o + (tm — tm) re ; 1 ‘The inequality 3 at ze (= «) (which follows from the Cauchy-Schwarz In- equality) implies that (ea = 2a)? ced Mom and f 2 2 ~ 2 22 am So. Lemna)’ = 81+8:2 umn! (og tam) 2 2 But a Yer & Pa Vlei tis)? Sn -6 a and the problem is solved. 32. [ Murray Klamkin ] Find the maximum value of the expression 7.2 +33 + see 2 jay + hay When 2,22,...,2n-1,%n 2 Oadd up to land n >2. Crux Mathematicorum Solution: First of all, it is clear that the maximum is at least a because this value is 2 1 attained for #1 = 3,22 = 3,23 =+++= 2" =0. Now, we will prove by induetion that 4 2p ah 2 2 Tag t thas tet eta t thet S oe y+ 105+ a (Old andl New Ineawalities a for all 21,,..-Tn-1,4q > O whieh add up to 1. Let us prove first the induetive step. Suppose the inequality is true for n and we will prove it for n +1. We can of course assume that zr2 = min{.r1,.r2,...2n41}- But this implies that hry + a3zy to + yey < (y+ yay tah toe +e tat yz +29) But from the inductive hypothesis we have 4 (et a2)Pag + afta toot th ate + h(E + 2) S 3 and the inductive step is proved. Thus, it remains to prove that a®b + Pe+@a< * ifa+b+e=1. We may of course assume that 2 is the greatest among a,b, ¢. Tn ths case the inequality a? + e+ea< (a+$) -(b-+5) follows immediately from abe be, 5 © a+ © theses . wwe have proved thata?b+Pe+@a< * and this shows that the maximum is indeed 4 oe 33. Find the maximum value of the constant ¢ such that for any 825.0458 qy7+° > O for which 2p) 1 > a1 +a -+++++ 24 for any k, the inequality Vit Vin te + Vin Seva Fata t Fin also holds for any n. IMO Shortlist, 1986 Solution: First, let us see what happens if rx, and a1 +:t+++-+-r, are close for any k. For example, we can take 14 = 24, because in this case we have zy +irot-+ +t, = ee 2, ‘Thus, we find that e> for any n. Taking the limit, we find that ¢ > 1+ V3. Now, let us prove that 1+ V3 works, We will prove the inequality Vit Vite t+ Vin <(1t+VOVn Fm Oo 48 Solutions by induction. For n = 1 or n = 2it is clear. Suppose it is true for n and we will prove that Ti+ Vta+--+ Vint Var < (1+ VO VH Fm Fa a Of course, it is enough to prove that Vin <(1+.V3) (Vater VA Fee) which is equivalent to Vata Pat Vat Peon S (14 VO Via. But this one follows because rita Sty Sane 34, Given are positive real numbers a,b,¢ and 2,9, , for which a += 1. Prove that Russia, 2002 1 = B)(1= 6) + ac + ab—a. Thus, abe+ xyz “all 5) Using these identities we deduce innmediately that val 3+ (xyz + abe) (4 3+ (aye + (z Now, all we have to do is apply the AM-GM Inequality e ,1+c,1-a Prove th ee ab First solution: We have the following chain of inequalities (Old and New Ineavalities 4 Second solution Because the inequality is homogeneous we can consider without loss of generality that a +b+¢=1 and so the inequality is equivalent to Lod Daas Ste so the inequality is equivalent to 1 1 Wehove y= 77 ro 1 Las Lagi thm ‘We will prove now the following intercalation: viet ties a S47 Tae ~ ‘The inequality in the right follows from the Cauchy-Schwarz Inequality: (Sh) (Ceen)z9 and the identity )7(a + 1) = 4. The inequality in the left can be written as )> ab < 1+ 9abe + a atl” Gabe which is exactly Schur’s Inequality. 36. Find the maximum value of the expression a@(btet+d+b(ctdtalt+e(d+atbj+dlat+b+o) where a,b,c, d are real numbers whose sum of squares is 1 Solution: ‘The idea is to observe that o%(b-+e-+d) +6%(c+d-+a)+c%(d+a+b) equal to > ab(a?-+b?). Now, because the expression ab(a? + (a), let us see how the initial expression can be written: Tobe +8) = ye Pate wt Taleo Her- Leo 7 Platb+cis ) appears when writing ‘The maximum is attained for a=b =e =d 37 [ Walther Janous] Let x,y, 2 be positive real numbers, Prove that z y z Se SSS eo r+Vetier) utVurawts) 2+Jetnety Crux Mathematicorum so Solutions First solution: We have (0+ y)(0-+2)=2y+(x? +: Hence DaytQeypetae = (VFT+ VE)? Layee SLs But ro ve Lanne Lee and this solves the problem Second solution: From Huygens Inequality we have Y=) +2) >a + Je and using this inequality for the similar ones we get — Og — tt EO o+Ja+ were) yt Jur auta) | 2+Vetnety) = y 2 +a va Now, we denote with a and the inequality becomes From the above notations we can see that abe = 1, 50 the last inequality becomes wring the denominators ab + be + ca > 3, which follows from the AM-GM Inequality. after cl 38, Suppose that a1 2. Prove that aya}+ aga ++ ana > axel + a Tran, 1999 Solution: A quick look shows that as soon as we prove the inequality for n =3, it will be proved by induction for larger n. ‘Thus, we must prove that for any a calc — a2) & (c8 — B)(ac — be) < (8 —a?)(ab — ac) Because a 0, which is clear. 39. [ Mircea Lascu ] Let a, b,c be positive real numbers, Prove that bte cta oth a ab e ). a > © —"\bye era” ate (Old andl New Ineawalities 8 Solution: 1 Using the inequality —— semen ey 2+ + weiner that ny aja 4 bb fo e — and Mf bre 1 be positive integers. Prove that at least one of the numbers «yi, °Y5,-.., °=-yn, "yz is less than or equal to 3. Adapted after a well-known problem Solution: Suppose we have a}, > 33 for all 4. First, we will prove that n? < 33 for all natural number n, For n = 1,2,3,4 it is clear. Suppose the inequality is true for n > 3 and let us prover it forn 7 ‘This follows from the fact that tebcithc Visa = 3.0822) nonst ‘Thus, using this observation, we find that ajt > 34> a" ais, > a, for alli, which means that a1 < a2 <+++ Bt? +26 & (2t-1)(t +1)? <0, 4 therefore 2¢ -1< 0.6 £< 3, thismeaning that 2y2 < 2 b) Denote also s = 2 +9 + =the following inequalities are wellknown (et+y+2) 23 (cy +22 +y2) and (ety +2) > Qtays; 82 Solutions then we have 28° > 5daryz = 27 — 27 (xy +22 + y2) > 27-95", ie. 2s" + 9s" — 27 20 @ (2s —3)(s +3)" 20, where from 2s ~ 32.0.4 > 3 1 Or, because p< g, we have 2239= (1-99) 23(1- if we put q=ay +22 + y2, p=ay2. Now, ome can see the following is also true aanteip>t 1 i ©) The three mumbers x,y, 2 eannot be all less than 5, because, in this case we get the contradiction 3.51 ay bas bye t Qepe< F425 because of symmetry we may assume then that 2> 4 We have 1 = (22 +1) cyt 2(e+y) > (22+1)zy + 2zy79, which can also be written in the form ((22 +1) y29—1) (7G +1) < 0; and this one yields the inequality 1 ws US Gy 2 We also have 1=(22 +1) zy +2(r+9) < een Eee sce ry) conse- quently ((22 +1) (e+) —2) (x+y + 2) > 0, whieh shows us that 2 trey ‘The inequality to be proved tial tatetoaetyss Sty (trys) can be also rearranged as +n (5-4) > ry _ (22-1) (2241) From the above calculations we infer that 1 a = Qe r+ ( B=) > 5 (et 2@2—NG2+3) (Old andl New Ineawalities sa (the assumption 2 > 5 allows the multiplication of the inequalities side by side), and this means that the problem would be solved if we proved 202243). 241 Rel = a 1 BAP + 62> 42? 44241; but this follows for and we are done. 1 4d) OF course, if z is the greatest from the numbers 2, 7,2, then z > 5; we saw that tatiatesy) = wr+n(3-4)> 2s 2(22~1) 22 +3) R41 where from we get our last inequality (2 s@tut)2 Bay Of course, in the right-hand side z could be replaced by any of the three numbers which is > 5 (two such numbers could be, surely there is one) aii tityt ry ‘Remark. Tes easy to see that the given condition implies the existence of positive nurnbers a,b,c such that x = —*,y And now a),}) and e) reduce ire" = aq! = ase immediately to well-known inequalities! Try to prove using this substitution d) 42. [ Manlio Marangelli | Prove that for any positive real numbers «,y,2, Baty + y22 + 2%2)(ay? + yz" + 22”) > ay2(e ty +2)" Solution: Using the AM-GM Inequality, we find that 1 we ay yar ve, > BT ere ay yet ae? bape YP Gat eet ey) Ge eae tay) and two other similar relations 1, te 329A Ea eT 1 x 2a? 30 AE dy, ty ee BP ee rey yes tay eet er try) Weta tae) ‘Then, adding up the three relations, we find exactly the desired inequality. of Sclutions 43. [ Gabriel Dospinescu ] Prove that if a,b,c are real numbers such that max{a, b,c} min{a,b,c} <1, then Leal tb ++ Gabe > 30% + 36% + 3a Solution: Clearly, we may assume thata = min{a,b,c} and let us writeb =atr,e=a+y, where 2t,y € [0, 1]. It is easy to see that a® +b? +e —3abe =3a(x?—ry+y?)+a° +y? and 0b + Be-+ a= 3abe = a(x? = zy + y®) +2%y. So, the inequality becomes 142° 4 y° > 32%y, But this follows from the fact that 14 2° + y° > 3zy > 3z°y, because 0< ay $1. 44, | Gabriel Dospinescu | Prove that for any positive real numbers a,b, we have @ e 2 tat at (+2) (+5) (+5) zoarb+g (tpt). Solution: By expanding the two sides, the inequality is equivalent to 2abe(a9 +09 +e + Babe — a2b — ate — Ba — Be — a — eb) + (02d + We + Fa? + 30°? = abe — abe? — abe? — ab?! —ab%e— abe?) > 0. But thisis true from Schur’s Inequality applied for a,b, cand ab, be, ca. a 48. Let ag 5 and ayy =an + Prove that 1-4 ae 2yntar > net and from this inequality and the previous one we conclude immediately that Ia y an 2 2 4 tan Atan Btan C(tan A+tan B+tan ©) > 3{tan A tan B+tan B tan C+tanC tan A) & © (tan A + tan B + tanC)? > 3(tan Atan B + tan BtanC + tan C tan A) clearly true because tan A, tan B, tan C > 0. 47. [ Titu Andreescu, Gabriel Dospinescu ] Let 2,y,2 $ Vand + y+ Prove that 1 1 1 7 Te ise < +2 = 10 Solution: Using the fact that (4 — 3¢)(1— 3¢)? > Ofor any # <1, we find that 12, Taw 8 @-2) ‘Writing two similar expressions for y and 2 and adding them up, we find the desired inequality, Remark. ‘Tough it may seem too easy, this problem helps us to prove the following difficult inequality +e-a 53 eww > rete In fact, this problem is equivalent to that difficult one. Try to prove this! 48. [ Gabriel Dospinescu ] Prove that if YF + /J-+ V7= 1, then (1-2? — yO 2)? 2 2 aye(e + uly + 2)e +2) Solution: We put a= V,b = Jando = Vz, Then1-2=1-a? =(a+b+0)?—a?= (b+ 6)(2a-+b +e). Now we have to prove that ((a+6)(-+¢)(c+ a)(a+b+e)(a+ b+ c)(at b+ 2))? > 2'5a°b*e? (a? + b?)(b? +e°)(c? + a”), But thisinequality is true se Solutions ato as it follows from the following true inequalities ab(a? + 62) < (this being equivalent to (a—b)* 20) and (2a-+b+e)(a+2b+c)(a+b+2c) > 8(b+0)(c+a)(a+8) 49. Let x, y, 2 be positive real numbers such that ry2 = a ay tyes ez Ast ys 2): (2) VE+ Vi+ VES SVR. r+ y+2+2, Prove that Solution: ‘The initial condition ry2=2 + y + 2 +2.can be rewritten as follows Now, let Then (1) We have aytystan > Mety+se Pte. cee, cee ath ath pee, Vv a(t ore, ost) 804i S E+ Bab 2 > abla tb) + be(b + 6) + calc + a) + Yala —b)(a-c) 20, which is exactly Schur's Inequality. (2) Here we have Vi + Vit VES VTE ® 7 te a3 [2], f=. = OV ive cra Vera arb ard p40 2 ‘This can be proved by adding the inequality =o t/a Vee with the analogous ones, 50. Prove that if «,y, are real numbers such that x? + y® + 2? =2, then ety teSaye +2 IMO Shortlist, 1987 (Old andl New Ineawalities st First solution: If one of x,y, 2 is negative, let us say x then 24 ayz—2-y-2=(2-y-2)-2(1-y2) 20, ety 2 because y+2 < YAPFF) < and 2y < O 1 we have 24 (ety) < VUE TW) Hey 1 Fay <2 + ay <2 + aye, ‘This ends the proof. Second solution: Using the Canehy-Schwarz Inequality, we find that atytenme=s(l-y)tyt2< Ve sus Fwy So, it is enough to prove that this last quantity is at most 2, which is equivalent to the inequality (2 +2y2)(2—2uz + (yz)?) <4 2(yz)> < 2yz)?, which is clearly true, because 2> 9? 4 2? 2 Que 51. [ Titu Andreescu, Gabriel Dospinescu ] Prove that for any #1,2,--.,1m € (0,1) and for any permutation o of the set {1,2,...,2}, we have the inequality ie -( Solution: 1 1 Using the AM-GM Inequality and the fact that —— < 7 write the following chain of inequalities at n 1 x 2) 7 F BS ss Solutions So, it is enough to prove the inequality Ya ia which is Chebyshev Inequality for the systems (2),:r2,...,2t,) and 1 tot aie ie 52. Let 21,.22,-..,tq be positive real numbers such that =1. Prove that Lvreo-vy Vojtech Jarnik First solution: Let —— =a;. Theinequality becomes Boks 2 Yala) (a1,02, Solution 2: With the same notations, we have to prove that woe <> ata (Old and New Ineavalities so Loa n (w-Dvna1- Va 2 VRE TRE Ji + Jit Ve Yin-1) = v and we are done. 53, [ Titu Andreeseu ] Let n> 3 and a1,¢9,...dy be real numbers such that 0;+09+...+0, >nand a?-+a2-+...4a2 > n?, Prove that max{ay,az,...,@n} > 2. USAMO, 1999 Solution: ‘The most natural idea is to suppose that a, < 2 for all i and to substitute 2; =2—a,>0. Then we have )“(2- x) =n ) 2; 0, we obtain )) 2} < (5 «) which combined with the above inequality yields 2) 2; < m). Thus, we have (n —4) (Sa) >0, which is clearly impossible since 1 > 4 and) 2; 2. 60 Solutions 54, [ Vasile Cirtoaje] If a,b,¢,d are positive real numbers, then b bee c-d d-a ord d 20. Fa atb= Cras ta ced" ab? Gd wwe get a=) b-c end d-a, sate) 4b+d) bee etd dta atb~(+e)+(dta) (etd tath Equality holds for a= ¢ and b = d. 4=0, Conjecture (Vasile Cirtoaje) If a,b,c,d,e are positivereal numbers, then did Fe 4 55. If and y are positivereal numbers, show that 2 + y# > 1 France, 1996 Solution: ‘We will prove that a! > ;, for any a,b € (0, 1). Indeed, from Bernoulli apb—ab Inequality it follows that a’! = (14 a=1)!-"<14(a=1)(1-8) =a+b—ab ion. Now, it 2 or y is at least 1, we are done. Otherwise, let, 0 < my < 1. In this ease we apply the above observation and find that a¥ 4 y* 2 y yay and thus the cone! my 56. Prove that if a,b,c > O have product 1, then (a+0)(b+0)(e-+a) > 4(a+b+e-1) MOSP, 2001 (Old andl New Ineawalities a First solution: Using the identity (a + 0)(6 + e)(e-+a) = (a+ b+ )(ab-+ be + ca)—1 we reduce the problem to the following one D4. atbre= ‘Now, we can apply the AM-GM Inequality in the following form ab tbe +cat 9 {lab be cae ab + be + het eas ety garde) And 50 it is enough to prove that (ab + be + ca)’ > 9(a+b +e), But this is easy, because we clearly have ab +e +ea > 3 and (ab+ be +ea)? > Babe(atb+0)=3(a+b+0) Second solution: We will use the fact that (a+6)(6+0)(e+a) > Z(a+b+e}(ab+be + ca) > 1. Using the AM-GM So, it is enough to prove that Flab + be-+ ca)+ Inequality, wecan write” +o4e 1 > atbre= Rad + be + on} + because (ab + be + ca)® > Babela +640) =3(a+b+0) 57. Prove that for any a,b,¢ >0, (2 +P +2)a+b-dlb+e—alle+a—b) < abelab-+ be + ca). Solution: Clearly, if one of the factors in the left-hand side is negative, we are done. So, we may assume that a,),¢ are the side lenghts of a triangle ABC. With the usual notations in a triangle, the inequality becomes 16K? ope? axbaes abelab+be+ea) + (a-+b-+e)(ab-+betca)R® > abe(a? +¥?+c"). But this follows from the fact that (a +b + ¢)(ab+be + ea) > Gabe and (@4P+2) 0 0. Provethat 5 g@t e+ 1)e+1) Btatbte — Kvant, 1988 Solution: ‘The inequality is equivalent to the following one Yor Dts pgoetht Da or Lipo pa abe Dat H+ Leet N252(Ca+ Las) But this follows from the inequalities be +2 > 2ab, tea + £ > 2he,cPab + 2 > Deu and 1 1 Wet 22a Par> > 50. [ Gabriel Dospinescu ] Prove that for any positive real numbers 21,2125--+4q with product I we have the inequality Solution: and ‘Thus, we have (Old andl New Ineawalities ea Of course, this is true for any other variable, so we can add alll these inequalities to obtain that which is the desired inequality. 60. Let.a,b,¢,d > Osuch that a+b-+¢ = 1. Prove that £842 ated > min {t 5+ x} Kvant, 1993 Solution: 1 Sin heey ae, The wave, aig it anon ha abe the inequality ne x ate) >a +42 — Z. We may assume that abe < Now, we will reach a contradiction proving that ©. +8 +84 abed> ; Tt is sufficient to prove that ath ee 1 r abe tat b+ e> poate But this inequality is equivalent to 4} >a" + I5abe > 1. We use now the identity Letale Ya? =3abe+1-3V ab and reduce the problem to proving that Jab < whieh is Sehur’s Inequality. 61. Prove that for any real numbers a, ,¢ we have the inequality Late P+ PP (a-o%b- 0)? > (1401+ P)(1+ 2)(a—H°(b—0)"(e—a), AMM Solution: avy cap ‘The inequality can be also written as 57 a 21 (of course, we may assume that a,b,c are distinct). Now, adding the inequalities (1+a7)(1+0?) | (1+ 6) +e) Cee * Fao > Fale (which can be found using the AM-GM Inequality) we deduce that (+e)0+8) 148? x= 2 praecal fee oP = ot Solutions and 50 it is enough to prove that the last quantity is at least 1. But it follows from 148 >| Lp aoa Me a)(b—e)] ~ pe and the problem is solved, 62. [ Titu Andreeseu, Mircea Lascu | Let a,2,y, 2 be positive real numbers such that ayz=1 and a> 1. Prove that a yt ae pre 2en ey 3 2 First solution: ‘We may of course assume that > y> 2. Then we have a prem rea aay and #9! Sy! > 2-1, Using Chebyshev’s Inequality we infer that Vee (le) (UA)- Now, all we have to do is to observe that this follows from the: inequalities 72°! 23 (Com the AM-GM Inequality) and > == > >} Second solution: According to the Cauchy-Schwarz Inequality, we have: lelss)eu(e2)+sterol (2+ + 2) 2 ( ute Fen ety tye)” ‘Thus it remains to show that (08 a 4H) ba (ey tye tn) Since (x +y + 2) > 3(zy + yz + 22), it is enough to prove that PB py Ry aty te From Bernoulli’s Inequality, we get 2H =[1+(e-D) and, similarly, ‘Thus a yy 4 (ety ta) (Old andl New Ineawalities os (ntyt2-3)> 1 Equality holds for ar Remark: 1 Using the substitution P=a +1 (922) andn= 2, inequality becomes as follows L 1 13 Fora Hera * aerh =? For = 3, we obtain one of the problems of IMO 1995 (proposed by Russia) 63. Prove that for any real murmbers 55... ny Yty +++ Such that 2}-++ ba = vite tah (tim — mm) <2 ( - San) . t Korea, 2001 \G)-E- (Ele <1, we find immediately that Solution: ‘We clearly have the inequality (m-ran)< YD @m— zu)? = ( Because we also have Som (-E)(ofo)ol-Eo) and the problems solved, 64. [ Laurentiu Panaitopol ] Let a1, @2,...,n be pairwise distinct positive inte- gers, Prove that ont 2 ay tanto an) aitajte tad TST Romania oo Solutions Solution: Without loss of generality, we may assume that a1 < az <++4< ay a; 2 4 for all i. Thus, we may take bj = aj— i> O and the inequality becomes oS n(n +1)Qn +1) ° 2nt1 n(n +1)(2n +1) Ye 2P a, Meewee sy Yop MOM En sD) oe 6 6 Now, using the fact that ai, > aj we infer that by < by < Chebyshev’s Inequality we deduce that nd hence MLSs, and the conclusion is immediate. Also, from the above relations we can see imme- if and only if a1,@2,.-.,dy is a permutation of the diately that we have equalit numbers 1,2, 65. [ Clin Popa ] Let a,b,c be positive real numbers such that a +b +: Prove that ofa av aVe-+ Vad) * Hi FBa + VO) * VBE a = Solution: Revrite the inequality in the form 1 With the substitution becomes zy + y2-+22= But, by applying the Cauehy-Schwarz Inequality we obtain ® (X)" s se 2s Lyra ein 2 where we used the inequalities (Eefsade d aye < Old and New Ineaualites or 66. Tit Andzeescu, Gabriel Dospinescu | Let a, 6, c,d be real numbers such that (140%) +8) +2)(14 #) = 16, Prove that =3< ab+be+ od +da+ac+ bd ~ abed <5. Solution: Let us write the condition in the form 16 = T](i+a)-[] (a—i). Using symmetric sums, We can write this as follows 16= (1-1 ha-Yab + FT abe + abcd) (146 a- YL ab- FT abe + abcd). So, we have the identity 16= (1 — Fab + abed)? + (32a ~¥- aby)®. This means that [1-5 ab + abed| < 4 and from here the conclusion follows. 67. Prove that (a? +2)( +2)(2 +2) > 9(ab + be + ca) for any positive real numbers a, b, APMO, 2004 First solution: ‘We will prove even more: (a? +2)(b? + 2)( +2) > 3{a+b+ ¢)?. Because (a+b +0)? < (al +[O| +|el)®, we may assume that a,b, care nonnegative. We will use the fact that if « and y have the same sign then (1+ :x)(1+y) >1+2+y. So, we write the inequality in the form n(ee+)2eee and we have three cases 4) Ifa,b,care at least 1, then T] sje! ii) If two of the three numbers are at least 1, let them be a and b, then we have m2) = (2X) (+P +N (P+ +E). atb+e? 9 a) by the Cauchy-Schwarz Inequality. If all three numbers are at most 1, then by Bernoulli Inequality we have and the proof is complete. o Solutions Second solution: Expanding everything, we reduce the problem to proving that (abc)? +2) a?b +4) 2+ 829) ab. Because 30a? > 3 ab and 2) 0°? +6 > 4° ab, we are left with the inequality (abe)? + S7 a? +2 > 27 ab, OF course, we can assume that a,b,c are non-negative and we can write a= 2,6 =y?,¢ = z?. In this case 2ab- Pale + yt Met yy te—alete-y) It is clear that if 2,y, are not side lengths of a triangle, then the inequality is trivial. Otherwise, we can take x = u+tv,y =v +w,2 = w+ wand reduce the inequality to ((u+ v)(v + w)(w + u))* + 22 16(u + v + wluw. We have ((u + v)(v + w)(w +u))'+1412>3/(uto tu (ut wy and it remains to prove that the last quantity is at least 16(u + + w)uvw. This comes down to Sy ey! 18 ay a (ut vv + w)Mw+ ul > S (wow) (a+ v +m) But this follows from the known inequalities (ut e)(ot wltwtu) 2 Sut et w)(ue tow +wu), (uv + ow + wu) > 3(uw)3, w+ 0+ w > 3eoR, Third solution: In the same manner as in the Second solution, we reduce the problem to proving, that (abe? +222 ab- oa? Now, using Schur’s Inequality, we infer that Sabe_ Yate 2 Lebo s ine and as an immediate consequence of the AM-GM Inequality we have Sabe <3 4/(abe?. atbtco— Yair} ‘This shows that as soon as we prove that abe}? +2> 3VlabeP, the problem is solved. But the last assertion follows from the AM-GM Inequality, Old and New Ineauaiin oo 68. [ Vasile Cirtoaje | Prove thatif0<2 1 then 2>y>x>1 andsso(1—2)(1- y)+(1- 2)(1—2y) > 0, impossible So we havear <1, Next we distinguish two cases 1) ry < 152) ay> 1 Day St Wehaves*y 1. From y > VF we get y > 1. Next we rewrite the relation e + y-+2= ayz+2 as x+y—2 = (ey —1)z. Because 2 > y gives e+y—2> (zy—Uy. (y= 12-2 ~ xy) > 0502 > (1+ y). Using the AM-GM Inequality, we have Ly > 2ypand ty = 14h ¥ > 3i/1-3-2, Thus we have 2 > 2ryy and 2 which means that ?y <1 and xy? < 2 ‘The equality xy = 1 takes place when x =y = 1 and the equality «*y? a 2 takes place when 2 = 3,y=2=2 69. [ Titu Andreescu | Let a,b, che positive real numbers such that a+b+c > abe. Prove that at least two of the inequalities 2 oP tS oe zs 8s Sog2a8+ sia >6, are true. TST 2001, USA Solution: ‘The most natural idea is to male the substitution > ‘Thus, we have 2, y,2>Oand ay +42 + 22 > 1 and we have to prove we fiat east no of the inequalities 2r+3y+6z > 6, 2y+32+6r > 6, 22 +32 +6y > 6 are true. Suppose this is ot the case. Then we may assume that 25+3y +62 < 6 and 2z+32-$6y <6. Adding hus, 12> — toytiit 0 Solutions which is the same as 12(y +2) > 5+9y? +82? + 12yz ¢ (22-1)? +(3y+22-2)? <0, which is clearly impossible. Thus, the conclusion follows 70. [ Gabriel Dospinescu, Marian Tetiva | Let 2, y, 2 >0 such that arty tz= aye, Prove that (e—1)(y -1)(z -1) <6v3-10. First solution: Because of x 1 (and the similar relations r2 > 1, ry > 1) at most one of the three numbers can be less than 1. Tn any of these cases (a < 1, y > 1, 22 Lor thesimilar ones) the inequality to prove is clear. The only case we still have to analyse is that when «> 1, y> Land 2>1 In this situation denote b, ‘Then a, b,c are nonnegative real numbers and, because Saya reatly=b+lz=c41, they satisfy atl+btlt+e+1=(a+1)(b+1)(e+1), which means abe + ab + ac+ be = 2. Now let «= Wabe; we have ab + ac + be > 3Vabacbe = 327, that’s why we get 29432? <2 (x +1) (29+ 22-2) <0 (0 +1)(e +14 V3) (© +1- V3) <0. For > 0, this yields Vabe =< v3 -1, or, equivalently, abe< (v3-1)', which is exactly (2 —1)(y—1)(z -1) <6V3-10. ‘The proof is complete. (Old andl New Ineawalities 1 Second solution: Like in the first solution (and due to the symmetry) we may suppose that 2 > 1, xy I; wecan even assume that x > 1, y>1 (for = 1 the inequality is plain). Then wwe get ay > 1 and from the given condition we have aty ay-1 ‘The relation to prove is (@-1)(y-1) (2-1) < 6V3-10 © Daye ~ (cy + 22 + yz) <6V3-9, or, with this expression of z, ry 4 — aye 4 68-96 © (ry — 2 —y)? + (6V3 ~ 10) ry <6V3 -9, after some calculations. Now, we put r=a+1,y=b +1 and transform this into a8 + (6V3-10) (a +5 + ab) —2ab>0. But a+b>2vab and 6V3 — 10> 0, soit suffices to show that oH? + (6v3 - 10) (2va5 + ab) ‘The substitution t = Vab > 0 reduces this inequality to t+ (6V3-12) # + 2(6v3-10) #0, 2ab > 0. or #+ (63-12) + 2(6v3-10) >0. ‘The derivative of the funetion f= #+ (6v3-12)1+2(6V3-10), 420 , 2 f wy=a(#- (v3-1) ) and has only one positive zero. Tt is V3 1 and it's easy to see that this isa minimum point for f in the interval [0, 0). Consequently £2 F(v3-1) =0, is and we are done. ‘A final observation: in fact we have F(0 = (vB +1)" (t+ 2v8- n Solutions which shows that f(t) > Ofor#>0. 1. [ Marian Tetiva | Prove that for any positive real numbers a,b, ¢, oa oe p+ -o+(e~ a? arb bee 7 Moldova TST, 2004 First solution: First of all, we observe that right hand side can be transformed into CSF = (Qh) e-21( (a= WMle- Hla~o) (Sab) EFFI) and so we have to prove the inequality (a= 1b o(e~a))(ab + be+ ca) (aFHG+ Nera) Tkis also easy to prove that (a+5)(b+e)(e+a) > x so weare left with ‘ 2 2 5 (Le-v) = {[Te-a]- Using the AM-GM Inequality, we reduce this inequality to the following one 8 3 w(X4) =[[e-5)- ‘This one is easy. Just observe that we can assume that a> b> cand in this case it becomes (a+ b+ e)(ab+ be+ ea) and (a-Dla-ab-o< and it follows from the AM-GM Inequality. Second solution (by Marian Tetiva): Itis easy to see that the inequality is not only cyclic, but symmetric, That is why we may assume that a > b > ¢ > 0. The idea is to use the inequality etryty [ar which is true if « > y > 0. The proof of this inequality is easy and we won't insist. Now, because a >b >c >0, wehave the three inequalities bo tabs th ec Pht arr 22 bee v z ate buts. b+ S04 b er and of course (Old andl New Ineawalities n ‘That is why we can write eae ap ab+e Pitete Da = OO te > @-H(6 $)+6-a(e+$)-@-o(a+$ = Le- a Tn the same manner we can prove that. peek (at b+) USAMO, 2004 Solution: We start with the inequality a — a? +3>a%+2 (a?—1)(a*-1) > 0. Thus, it remains to show that 4 TIle+2) > (Sa) Using the AM-GM Inequality, one has a 1 1 3a Ben" WR TR Tea We write two similar inequalities and then add up all these relations. We will find that 5 The +22 (Xe). which is what we wanted. 73. [ Gabriel Dospinescu ] Let n > 2 and 2}, 2,..-,2, > Osuch that Prove that um Solutions Solution: In this problem, a combination between identities and the Cauchy-Schwarz Inequality is the way to proved, So, let us start with the expression (+22), (+22) = 5 (443 acicten V7 - G)& ‘Thus we could find from the inequality (2+2 istejen S50 TE (&)(E3) Unfortunately, this is not enough. So, let us try to minimize ‘We can immediately see that isteten that ‘This could be done using the Cauchy-Schwarz Inequality’ x (E+ (eens bali) rsigfen S41" ( Because > (é a ) 1, we deduce that (S*)( Janta which is what we wanted to prove. Of course, we should prove that we cannot have equality. But equality would imply that 21 = 2 = ++ = a, which contradicts the a ()E8)- Old and New Ineauaiin % 74, [ Gabriel Dospinescu, Mircea Lascu, Marian Tetiva | Prove that for any po- sitive real numbers a,b,c, EPH E+ Dabe+3> (1+ a1 +O/(1 40) First solution: Let f(a,b.c)=a? +2 +2 +abe+2—a—b—c—ab~be— on. We have to prove that all values of f are nonnegative. If @,,¢> 3, then we clearly have : + i +t <1, which means that f(a,0,¢) > a?+ #4 e+ 2—a—b—e> 0. $0, wemay assume that a: (m+n — 1a" gpminel_ymtnal gm ym gn a (m+n—Dle—y) © m(e—y) ley) ‘we have assumed that x > y). The last relation can also be written h ed th ‘The last re also be wri eon fein [md fo and this follows from Chebyshev's Inequality for integrals. Vz" —y") ‘77. Let a,b,c,d,e be positive real numbers such that abede= 1. Prove that asabe b+ bed etede d+ dea eteab 10 abt abed” T+ bet bade T¥od+ eden” T+de+deah T+easeake~ 3° Crux Mathematicorum Solution: ‘We consider the standard substitution ad 7 (Old and New Ineavalities ” with x,y, 2,4,u> 0. tis clear that at abe T$ ab + abed Writing the other relations as well, and denot a5, we have to prove that if a; >0, then x Using the Cauehy-Schwarz Inequality, we minor the left-hand side wit 4s? Wat ala (as oe = Tata? where S$ =)” a;, By applying the Cauchy-Schwarz Inequality again for the denominator of the fraction, we obtain the conclusion, '78.[ Titu Andreescu ] Prove that for any a,b,c, € (0 3) the following inequality holds sina -sin(a— 6)-sin(a—¢) , sinb-sin(b— c) -sin(b—a) , sine-sin(e— a) -sin(e~ 6) —sinfo+e) SOS ime a) SSC) TST 2003, USA 20. sin(b Solution: Let 2 =sina, y=sind, 2 =sine. Then we have, y,2> 0. Tt is easy to see that the following relations are true: sina -sin(a —b)-sin(a —¢) -sin(a +8)-sin(a +e) = a(2? — y?)(z? — Using similar relations for the other terms, we have to prove that Yae?- ye? - 7) 20. With the substitution 2 = Vu, y = V0, z = Vi the inequality becomes > yi(u— ‘v)(w—w) > 0. But this follows from Schur’s Inequality. 79. Prove that if a,b,c are positive real numbers then Vr + VPP a PAF Oa > Varb+ Het a+ Val Fb + at KMO Summer Program Test, 2001 Solution: It is clear that it suffices to prove the following inequalities Dots Dots Neos Dat! cy Solutions (Le) (Le) (L)(LH)- ‘The first one follows from Schur’s Inequality Yeats abe a> Vi a°o+ Ya? Vet S abe a. ‘The second one is asimple consequence of the Cauchy-Schwarz Inequality: (a8b+ Bet cha)* <(a°h + be? + ca?)(a' + b+ c8) (ab? + be + 0°)? < (a8 + Be + Aa?\(at + + cA) and and the fact that 80. [ Gabriel Dospinescu, Mircea Lascu ] For a given n > 2 find the smallest constant k,, with the property:if a),...,a,>0 have product 1, then aay a2 EF aed = a} * WF asa} * Solution: Let us take first ay = a2 = = Ay. Weinferthat 2 2 2 Gere Nema” Urey Wray for all x > 0. Clearly, this implies kn > n —2. Let us prove that n — 2 is a good constant and the problem will be solved. First, we will prove that (x? + y)(y? + x) > «y(1+«)(1+ y). Indeed, this is the same as (x + y)(x — y)* > 0. So, it suffices to prove that. 1 1 1 Tralee) * Trayiea) tT reead Now, we take ay = = © and the above inequality becomes r2 r : Tea teg2 1-2 L(t)? which can be also written in the following from Sa+b+e\ptgrn), (a+p)?+(b+q)? +(c+rP > Farb+ap+a+n) Since A(a+b+elptq+r)<[latb+e)+(p+qtrl, it suffices to prove that (tpt b+ oP +(ctr? > Hat +O+g (ern? ‘This inequality is clearly true. 80 Solutions 82. Vasile Cirtoaje | Prove that the sides a, , of a triangle satisfy the inequality able ble a a(g+2+5-1) pa(2sge2). First solution: + We may assume that ¢ is thesmallest among a,b,c. Then let 2 = b— = After some computations, the inequality becomes (8a-2o2*+ (2 +e~ 4) (a0) 20 # (Ba-2)(26-a-c)?+(4b+2e~Ba)(a-c)* > 0 which follows immediately from 3a> 2c, 4b + 2c — 3a =3(b+e~a)+b-e>0. Second solution: Make the classical substitution a =y + =,b=2+st, nators. The problem reduces to proving that = +yand clear denomi- ag yt et 4 2aty tye + ete) 2 Mey? + ys" + 22”) We can of course assume that «is the smallest among x,y,z. Then we can write y=x-+m,z=2 +n with nonnegatives m and n. A short computation shows that the inequality reduces to 2x(m?— mn +n?)+m 4 n° +2m?n ~ 3n?m > 0. All we need to prove is that m3 +94 2m?n > 3n®m < (n—m)§—(n—m)m? +m >0 and this follows immediately from the inequality f° +1 >3, truefor t> —L 83, [ Walther Janous ] Let > 2 and let ,72,...,7, >O.add up to 1. Prove that i102) =f). Crux Mathematicorum First solution: ‘The most natural idea is to use the fact that nomi, n= I= Bp at tata tee ‘Thus, we have (2) -0- =) and we have to prove the inequality (+2) 210 =): (Old and New Ineavalities a But this one is not very hard, because it follows immediately by multiplying the inequalities T(1+4) 2 ( ‘ was 5 obtained from Huygens Inequality. Second solution: ‘We will prove even more, that TI(s ‘This follows from Jensen's Inequality for the convex function f(x) In(1— 2), So, it suffices to prove that Ts But a quick look shows that t the problem 121 In(1+ 2) — tle s is exactly the inequality proved in the solution of 84, [ Vasile Cirtoaje, Gheorghe Eckstein ] Consider positive real numbers 21, Poy on By Such that 24.79. —1_ n1Fa 1. Prove that yt mim TST 1999, Romania <1 First solution: Suppose the inequality is false for a certain system of n numbers, Then we can find a mamber & > 1 and n numbers which add up to 1, let them be aj, such that = kaj. Then we have Tee) (n= 1) Yi Badan Second solution: Let us write the inequality in the form lta ~ n +22 * tin ‘This inequality follows by summing the following inequalities 14 n a natn ah te tee ‘The first from these inequalities is equivalent to BO pal Fa tal > (m—1ay* and follows from the AM-GM Inequality. ‘Remark. 1 1 Replacing the numbers 2,22,...,, with + nm hh respectively, the in- equality becomes as follows " 1 1 L —— 1. Teme Tem de Teme, = 85. [ Titu Andreesen ] Prove that for any nonnegative real numbers a,b,c such that a? + 6+? + abe =d we have 0 3V@70E and thus it is enough to prove that abe < 3@0%e?, which follows from the fact that abe < 4. The upper bound instead is hard. Let us first observe that there are two numbers among the three ones, which are both greater or equal than 1 or smaller than or equal to 1. Let them be band c. Then we have 4> Qbe + a? + abe > (2 — a)(2 + a) > be(2 +a) => be <2—a ‘Thus, ab-+be-+ea—abe 0, whieh is true due to our choice, (Old andl New Ineawalities 88 Second solution: ‘We won't prove again the lower part, since this is an easy problem. Let us con- centrate on the upper bound. Let a > b > ¢ and let a = 2+y,b = 2—y. The hypothesis becomes (2 + ¢) + y®(2—¢) =4— 8 and we have to prove that (2? = y*)(1— 0) < 21 = 20). Since y? = 2+ ¢— 5 *"2", the problem asks to , =u prove the inequality 4° GE) 4) < 211 ~ ca). Of course, we have e < 1 and 0 < y? = 240 Bert & a2 < 2-0 = 2 < VIR6 (we have used the fact that a > b = y > 0 and b> 0 = x > y > 0). Now, consider the funetion f : [0,v2—¢] > R, f(x) = 2(1- er) - bao) We have f(a) =—2e—8e = Oand thus f is decreasing and f(x) > f(v2— 0). So, we haveto prove that S(VI=8) 20 A(1-evI—0) > (2-e)(1-c) @ 3B ch 2VI=C @ (I-VI oF FO clearly true. Thus, the problem is solved. 86. Titu Andreescu ] Prove that for any positive real numbers a,b, the following, inequality holds Ae | ake a+ b+e—Vab—Vvbe— Vea). Now, we will prove that a+b+e~3Vabe <2(a+b+e~Vab—~ vbe— Jaa) and the problem will be solved. Since the above inequality is homogeneous, we may assume that abe = 1. Then, it becomes 2Vab + 2Vbe + 2/ea—a— b—c < 3. Now, using Schur’s inequality, we find that for any positive reals x,y,z. we have: a2¢ 8 cayman “ature All we have to dois take x = a, y = VB, z = Vin the above inequality, a 2oy + 2y2 + Qex a Solutions 87. [ Kiran Kedlaya | Let a,b,c be positive real numbers, Prove that oA VBA «of SEE SHI Solution (by Anh Cuong): We have that a+ Vab+ Vabe &. Vietnamese IMO Tr: ing Camp, 1995 Solution: We will prove that 7 is the best constant, We must clearly have k < (1+ VED) [sn eV) [forall postive integers. Because [sin (VT) |= sin 7, we deduce that — > sin —2— > —4_ from i+ Vee DL bee a FFA” where it follows that & < 5. Now, let us prove that this constant is, soo! Clearly, the inequality ean be writte (va) > 5 + A) We have two cases (Old and New Ineavalities as 1 4) The frst caseis when {Yl} < 4. OF course 1 (Vi) 2 Vii VT = es and because sinc > — = we find that Let us prove that the last quantity is at least ——"——. This comes down to 2 ws 2+vi levi ° 3s =) or 6( i+ Yn = 1)? + 3( Yn + Vn 1) > w"(1 + i) and it is clear. Let 2 = 1={ ya} < # and let n = ii) The second case is when { Yl} > 1 K+ p< p< 2k, Because {Yi} > 5 + p > k+1. Then it is easy to see that ' 3 rz Fu VSR and so it suffices to prove that yl + VES OR - + VER Using again the inequality sin > 2-75 we infer that, 2VETEE— VETTE h+1 2 1+ VR eR 3(1 But from the Cauchy-Schwarz Inequality wehave 2/74 R — VFR —k > 0. Because the inequality (14h + VTF2R)’ > 5 (1+ VIEFR) holds, this case is also solved, 89. [ Dung Tran Nam ] Let 2, y,2> Osuch that (1 + y + 2) =32zy2. Find the ini aitytt et Yajninna and main of 5S @ryte ‘Vietnam, 2004 First solution (by Tran Nam Dung): and zy2=2. Thus, we have to find the ‘We may of course assume that 1 +y +2=: 86 Solutions bg ylaet extremal values of oe Now, we have sttytet = (tty toy —2p aya = (16-2) ay)? = 2°00 ay)? + Aaye(e ty +2) = = 2a? Gla + 288, 2 where a = ay +ye-+2r. Because y+2 = 4—z and yz = >, we must have (4-2)? > = which implies that 3-5 4 ortoce [ | But this reduces to the study Bove of the function f(z) = 42? — 82 i defined for [ | which is an easy task. 90. [ George Tsintifas ] Prove that for any a,b,e.d >0, (a+ G+ c+ d)(d +a)" > 6a AP(a+b+e+ Ad Crux Mathematicorum (Old andl New Ineawalities aT Solution: Let us apply Mac-Laurin Inequality for ibe, = bed, z = eda, t = dab, We will find that Yate bed eda ad Da Tr ‘Thus, it is enough to prove the stronger inequality (a+ G+e(e+ Hd +a) > (a+b-+e+ d(abe-+ bed + eda + dab) Now, let usobserve that (a+ (0 + o}(e-+ d)(d +a) = (ae-+ bd + ad + be) (ac + bel + ab + ed) = (ac + bd)? + Ya (be + bd + edd) > Aabed + Fa (be+ bd + ed) = (atb+e+ dlabe-+ bed + eda + dab), And so the problem is solved. 91. [ Titu Andreescu, Gabriel Dospinescu ] Find the maxinnua value of the ex- pression (aby, (bey"® | (cay t=ah*1-h* imam where a,b,c are nomegative real numbers whieh add up to 1 and n is some positive integer. Solution: First, we will treat the ease n > 1. We will prove that the maximum value is It is clear that ab, be, ca < i and so 1 pa (aby (be) lah ~ be LM Fay" +)" +(ca)") ‘Thus, we have to prove that (ab)" + (be)" +(ea)" < z Let a the maximum among «a,b, ¢, Then we have di Daa)" =a"(b +e)" >a" + ac" + nab" le > ab" + bc" + cha". So, we have proved that in this case the maximum is at most — yopet a > FET for n> 1. Now, suppose that m= 1-Tn ths ease we have a 1 108{abe)? + B8abe <1, 7 1 which is true because abe < Hence for n = 1, the maxima value is 2 attained fora =b = 92, Let a,b, be positive real numbers, Prove that 1 L 1 a0) H+” Fa) 3 Vabe(1 + Vabe) Solution: ‘The following observation is crucial (1 1 _ a aba (Tet => = La ‘We use now twice the AM-GM Inequality to find that 3 Lea, lel) Ym Lary Dias 2 ya And so we are left with the inequality 3 3Vabe— 3 Tr abe Fabel + Vabe) which is in fact an identity! 93. | Dung ‘Tran Nam | Prove that for any real numbers a,b, such that a? +6? + 9, 2(a+ b+ 0 ~abe < 10. Vietnam, 2002 (Old andl New Ineawalities a0 First solution (by Gheorghe Eckstein, Because max{a,b,c} < 3 and |abe| < 10, it is enough to consider only the cases when a,b,¢ > Oor exactly of the three numbers is negative. First, we will suppose that a,b,c are nonnegative. If abe> 1, then we are done, because Yat b+) ~abe<2VHAFPFA)—1< 10. Otherwise, we may assume that a <1. In this case we have z nierhsd aoc 2(o42 [Fee =2a+2V18— 20? < 10, Now, assume that noi all three numbers are nonnegative and let ¢ <0. Thus, the problem reduces to proving that for any nonnegative 2,y,2 whose sum of squares is 9 we have d(x +y = 2) + 2ry2 < 20. But we can write this as (2—2)?-+(y-2)?+ (2-1)? 2 Qaye — 62 — 2. Because 2ry2—62—-2 < a(y?+2)—G2—2= =23432—2=—(2 -1)%(2 +2) <0, the inequality follows. Second solution: OF course, we have |), |4|, |e] < 3 and Ja+b+el, abel < 3V3. Also, we may assume of course that a,b,c are non-zero and that a 0. Ifa <0 3a — 1.4 (a +1)2(a —2) <0, whieh follows. $0, we just have to threat the case 0 1 we have b,c > 1 and the inequality is again. Thus, the problem is solved. Jes 94, [ Vasile Cirtoaje ] Let a,b,¢ be positive real numbers, Prove that (a+ f-1) (0 4-2)(o44-a)(eebr)e(ont-s) (oof First solution: 1 1 1 With the notations r= a +5 — Ly =b+o-1,2=e+- = 1, the inequality becomes ry tyetor>3. We consider without loss of generality that 2°= max{r,y,2}. From 1 tad 1a (+ 1)(yHl)(2+1)= abet F tatbtet +547 > Qtatbtets +p Stety+z, 90 Solutions we get aye tay tyz +2024, with equality ifand only if abe = 1, Because y-+2= ¥ two cases a) x>0, yz <0; b)x> O,y>0,2>0, a) x> 0, yz <0. We have xyz < 0 and from ayz + ry + yz tar > d we get ay tyzt2r>d>3 b) x> O,v> 07> 0, We denote ay + yz + 2: inequalities we have > Owe distinguish 3d? with d > 0, From the mean ay tyst D3 PPA, from which we deduce that xyz < d°, On the basis of this result, from the inequality ayzt+ay+yzt er > 4, we obtain d* + 3d? > 4, (d—1)(d +2)? > 0,d > 150 ay + yz + 20 23. With this the given inequality is proved. We have equality in the case a Second solution: Letu=2+lu=ytl,w=2+1. Then we have 1 uw =u+o+w+ aber > Now, consider the fumetion f(t) = 2t8+¢?(u+v+w)—unw. Because im f(t) =00 and $(1) <0, we can finda veal mumber » > 1 such that f(r) = |. Consider the numbers m=4n 2 they verily mnp =m +n-+ p +2 we deduce from problem 49 that mn+np-+pm S2(m-+n-+p) Suv +ow+wu>2r(u+v-4u) >%Au+v+w) But because u = x+1,v = y+l,w = 241, this last relation is equivalent to ay + yz + 22 > 3, whieh is what we wanted. utvtws2 95. [ Gabriel Dospinescu ] Let n be an integer greater than 2. Find the greatest real number m,, and the least real number M, such that for any positive real numbers ByFaye soya (With y= 204 2n41 =H) Solution: 1 We will prove that mn = sys Laan tn (Old and New Ineavalities a is trivial, because 2; +2(n—1)2¢+ ais1 < 2(n—1) +) ag for alld. This shows ei that my > 5A. Taking 2) = 2 the expression becomes 1 (n-22 TF Rn—) TF Qn dete THO and taking the limit when a approaches 0, we find that my < 5 ma= st 2n=1 Now, we will prove that M, > 2. OF course, it suffices to prove that for any Tyas-royn > Owe have 2 1 ne! TF 2(n= Da Fai = 2 But itis clear that Daven datan <> ees aj, we have to prove that if [fa = 1 then ‘Taking DEE 0 29, b+ be + ca. After making or (3d —1)? + (14d + Qabe) > 0. which is Schur’s Inequality. 97. [ Vasile Cirtoaje ] For any a,b,c,d > 0 prove that 2a +1) + IE + IE +1) B (1+ abed)(1+ a )1+ PYA+ A) +e). Gazeta Matematica Solution: Using Huygens Inequality TI] +64) = (1+ abeay' we notice that it is enough to show that that YT] +t [Ja +e) +0?y4 Of course, it suffices to prove that 2(a° + 1) > (a4 + 1)(a? + 1)* for any positive real a. But (a?+1)' < (a+1)(a? +1)? and we are left with the inequality 2(a7+1)? > (a +1)(at+1) 4 2(a?-a+1)? >at+1 © (a—1)'>0, whieh follows, 98. Prove that for any real numbers a,b,c, (a+ de b4o't(e+a)'> fat soey Vietnam TST, 1996 (Old andl New Ineawalities 28 Solution: Let us make the substitution a +b = 2z,b + 2r,c + a = 2y, The inequality becomes So(y +z 2)‘ <$ 28) 24. Now, we have the following chain of identities Dts = DT (Le 4 dye - 209-222) =3(Tx4) H4(Ce) (Coz- 2-22) 4 Dey +299" =3(D2) —4(Na) (Le) + +16 Sey" 4(Sav) =4(e)' Hv (Lz) 3, y > 3,2? > By, we have bey 252°, SE > yey? > Or, Say > 12.29 > Sy and a*y > 2. Summing up these inequalities, the desired inequality follows, ‘s s son tn 223 100. [ Dung Tran Nam ] Find the minimum value of the expression = 45 += where a, b,¢ are positive real numbers such that 21ab + 2be + Be < 12, ‘Vietnam, 2001 First solution (by Dung Tran Nam): Lett == 2,>=y,— ==. Then it is easy to check that the condition of the problem becomes Dry2 2 2r + fy + 72. And we need to minimize 2 + y +2. But Qry >7 20+ dy Qry = 7 sa -nzaesunf . a Solutions Now, we transform the expression so that after one application of the AM-GM Inequality the numerator 32y—7 should vanish at y-+2 2 a-ha SM = i 7 Ut u AL E>etealy Ty 3te 3 9515 ‘i that 24 142 pt andsor ty +22 gttt2y ‘We have equality for r= 3,y = 2,2=2. Therefore, in the initial problem the answer is “achieved for i But, it is immediate to prove Second solution: ‘We use the same substitution and reduce the problem to finding the minicwum value of x+y +2 when 2ry2 > 2r+4dy +72. Applying the weighted the AM-GM Inequality we find that seye2 ()*on8(#8)*. And also 2r-+ 4y +72 10) «124 5% «ct .yt- 2%. This means that (2+ y+ 2a + 4y +72) > Pays, Because 2ry2 > 2x + dy +2, we will have 1 (ety+ srtytret with equality for r= 3,y 101. [ Titu Andreescu, Gabriel Dospinescu ] Prove that for any, y,2,a,,¢>0 such that ay +yz+27=3, a ’ c ree tt ett Get wes. Solution: ‘We will prove the inequality b c te D Veter y—(ety +2), are ret ats which may be obtained by applying the Cauchy-Schwarz Inequality, as follows (e+2)+ Gets atbte) (atb+e) at Lvewmess ‘o show that EAD) aot ye) > $VTFEt VET E+ VETO (etyt Ar+y+2) A good exercise for readers is Yi Vet Mes 224 y 42+ Veuve Fae), 102. Let a,b, be positive real numbers. Prove that (b+e-a)? | (c+a-b)? Grae (a+b—e)? Using the Cauchy-Schwarz Inequality, we find that (ey (@ty+2-3) Ty ea3 90 Solutions and 50 it is enough to prove that 3s (Ne - Le +3 Day +1820. But from Schur’s Inequality, after some computations, we deduce that Sy 22) x. Thus, we have (Sa) Yara Nays is> (S2)'-9 Ler i820, the last one being clearly true since )> 2 > 6. Second solution: Of course, wemay take a-+b-+¢=2. The inequality becomes 4-a? 3 1 7 Uppa-a25 ° Lipase 0 then at Fan-1 . siete.) attap te > 8 — nares an 2(n—0 ( where ay is the least among the numbers a1, 2-54. Solution Let a; an = 2; 20 for ie {1,2 sn ~1}.Now, let us look at asa polynomial in a= a,. Tt isin fact, Tle+s-@-(2 We will prove that the coefficient of a is nonnegative for all k € {0,1,....m—1}, because clearly the degree of this polynomial is at most n — 1. For k= 0, this follows from the convexity of the function f(x) os Sor n)"=na (Old andl New Ineawalities or For k > 0, the coefficient of at is (Ss TS emensk a 1Siy cng k SAT Let us prove that this is nonnegative. From the AM-GM Inequality we have n x Tiyan Pigg S ieischne Sncase-t aE tentensert)= (QE & SEs Sig cSig eM which is clearly smaller than (:) Yat", This shows that each coefficient of the polynomial is nonnegative and so this polynomial takes nonnegative values when restricted to nonnegative numbers, 104. [ Turkeviei ] Prove that for all positive real numbers x, y,2,t, cht yte ty + omypt ay ays PS Pe bat PP Kvant Solution: Clearly, it is enough to prove the inequality if ay2t = 1 and so the problem becomes T.a,b,e,d have product 1,then a? +6240 +d2+2 > ab+ be+ ed +da+ ac+ bi Let d the minimum among a, b,¢,d and let m= Yabe, We will prove that PLR E+E 42 (ab + bet ed + da tact bd) > d+ 3m? +2 —(3m? + 3md), whieh isin fact +B 4e—ab—be- cad (atd+e-3Vabe). Because d < Yabe, proving this first inequality comes down to the inequality a+b = ab be—ca> Vale (a+b+0~3Vahe). ‘Take u= ae Using problem 74, we find that _ Yate” ~ Yabo’ = who tuts 3> udu tw tut ow + wu which is exactly a? +08 +2 —ab—be= ca > abe (a+b+e~3WVabe), Thus, it remains to prove that d? 42> 3md & d?4.2>3V@, whiehis clear 98 Solutions 105. Prove that for any real numbers a, 2,...,d, the following inequality holds Solution: Observe that ay -ys 1 itd dat 0) = ye is [ d= [ (Seine) (te) Now, using the Cauchy-Schwarz Inequality for integrals, we get LE) (C Emre) (Es), which ends the proof. 106. Prove that if a1,a2,...,4y,b1,..-.0, ave real numbers between 1001 and 2002, inclusively, such that a}+a3+++++ a2 = 3+ 6+++++ 62, then we have the inequality TST Singapore Solution: 1 ‘The key ideas are that $* [ie for any d and that for all x € [be] we have the inequality 2? +1< be, Consequently, we have 5a Beale > Sabi > a +0 and also FL ewz Ll (Old andl New Ineawalities 20 a ao . «Bi a? Now, the observation that Bach and the inequality Db, lt B allow us to vite Sa? > 7 2 4 ajb; and adding up these inequalities yields a BS =o +abs) Le ‘Using (1) and (2) we find that al a iz Ad pa nto, Sto which is the desired inequality. Lune 2) (aj + ag tee tan)s 107. [ Titu Andreescu, Gabriel Dospinescu ] Prove that if a,0,¢ are positive real numbers which add up to 1, then (PHPYVFVE +a) > + PE + ea)? Solution: Leta 1 # Ge? ty yi + Pe? +2") > 8a? ty? + FP ‘We will prove the following inequality 2. We ind the equivalent formnif 2+ 4 2=1 then Gra er+yers2) (4242) aeetey sey for any positive numbers 2,9, Write 2? + y? =2c,y? + 21 2a, 2742 =2b, Then the inequality becomes Dyes Recall Schur’s Inequality Vat + abela b+) > wld +e) abela+b+o> Wraklb+e~a). Now, using Hélder’s Inequality, we find that ‘1 - @ (oy) Lelbte-g=V——— a 2 > a=) Cae Combining the two inequalities, we find that x treca 2D and so the inequality is proved. Solutions 10 108. [ Vasile Cirtoaje ] If a, b,c, dare positive real numbers such that abed = 1, then 1 14 5 Trae oF ree rae Gazeta Matematica Solution: If follows by surnming the inequalities 1 1 Ta ia ze 1 L 1 (sp sap TF ‘The first from these inequalities follows from L 1 1 abla? +12) ~ a6? ~ 2ab rer oF Tea ~ Farrer ter ab(a— 6)? + (ab— 1)? 1 oP HPT + dy = =1 Equality holds if a= 6 = eo Gazeta Matematica Solution: ‘We have the following identities a a _abla—t)+aca—0) Ore se ab(b—a) =F ‘Thus, we have ¢ ay ]_ale—b)_—_allob) Ure Del eee ee = (2eRads oa) abla)? = (ash abt beet) Tapas NTT (Old andl New Ineawalities 101 110. [ Gabriel Dospinescu ] Let a1,42,.-.s4n be real numbers and let S be a non-empty subset of {1,2,...,n}. Prove that (Ee) < Y ws tay? a8") Pen TST 2004, Romania First solution: Denote sz = a1 +-++ +0, for k= T,7i and also sy, = 0. Define now 1 ifies 0, otherwise Using Abel's summation we find that Vai = arbi + arb tort abe = s b, = 81(b1 — by) + S2(bp — bg) ++++ + 8-1 (Pn-1 — bn) + Sabin + Su4a(—bi) 1. So Now, put b: ~ bs = a1, bo — bs = m2,-+. by = we have = Ipntsbe = Fae Tnt = Ya=F as, a and also x; € {-1,0,1}. Clearly, }* 2; = 0. On the other hand, using Lagrange identity we find that =H YS teeta)? = V+ VY (y-syt= sgigicn Bi actin = Y w-s=m+n yy 1sigfenht a So we need to prove that won a2 (Sas) «(B4) But it is clear that (Soe) +(¥ ) Sga+e) iI = + 2 SO sisj(xi2j +1) 10, we can write 2° DY sislets YO Wis 8)0tne)= asin 1siEntt n (sp teeth y,) 4 shri (ap bag te tan) tet stealer t = Shak + n(si ter + 8h). bebe tees) =e (a0) +(¥ ) < Yeauien+Ea0-a= t mi pet =(n +1) ¥° sf and we are done, a Second solution (by Andrei Negut): First, let us prove a lemma Lemma For any a), @9,...,2¢41 € R we have the inequality ‘ 2 (Se) < YO @t- taj)". = 1eicieent Proof of the lemma Let ns take si = a1 +++ +p. We have ‘ Ye ace = 81+ 89> Sob Saks — Soe oi and so the left hand side im the lemma is bt DsF+2 YL sain rsaiert+ 2 SY s2i82)-2 D7 s2ieis24 o wow > 2ab and thus we have (a; + ay +++ + a4)?-+ (a2 tas +-+-Fa_-1)? > 2arag é » ‘Also, the inductive step for ag,-..,ay shows that (a; ++ +5). Xn, <2, So, it suffices to show that x ab s20 as S (atta) Slate tay? iy St a But thisis clear from the fact that a? appears in the right hand side and by summing up the inequalities from (+). 111. [Dung Tran Nam J Let 21,9... 22001 be real numbers in the interval [—1, 1] such that 2}+03+...+-fo9, =0. Find the maximal value of the 2 +2 +-+-+ 22008 Solution: 1 Let us take a; = 29 and the funetion f :[-1,1] +R, f(«) = 23. We will prove first the following properties of f: 108 Solutions 1 flety+l)+f(-1)2 fla) + sit -1<2,y <0. 2. f is convex on [1,0] and concave on [0,1] 3.1 2>Oandy Othen f(x) + f(y) < f(x + y). The proofs of these results are easy. Indeed, for the first one we make the substitution x = —a°,y = —6 and it comes down to 1>a°488+(1-a—b)? 4 1 > (a+b)(a?—ab+b*)+1-3(a+b)+3(a+b)?—(a+b)> & 3(a+b)(1—a)(1= 5) > 0, which follows, The second statement is clear and the third one can be easily deduced in the same manner as 1 From these arguments we deduce that if (t), t2,...,t200¢) = t is the point where the maximum valueof the function 9 {re [-1, 1PM |r +++ +t, =O} 200 Bg(er.-.-,22000) = > f(y) (this maximum exists because this function is ma defined on a compact) is attained then we have that all positive components of tare equal to each other and all negative ones are —1, So, suppose we have k components equal to —1 and 2004—f components equal toa number a. Because th +ty-++++toone = O wend that a= 5 A and the value of gin this point is (2004—) ky, ‘Thus we have to find the maximum value of (2004 ~ 8) {/ 5 a z the set {0,1,...,2004}. A short analysis with derivatives shows that the maxirmumn is attained when k = 223 and so the maximum value is Y223 - V17SF? — 228. — k, when kisin 112. [ Gabriel Dospinescu, Clin Popa ] Prove that if'n > 2 and ax,03,.-.44y are real numbers with product 1, then 2n Ohtabee te at—n> 2 tay Solution: ‘We will prove the inequality by induction, For n= 2it is trivial. Now, suppose the inequality is true for n—1 numbers and let us prove it for n. First, itis easy to see that it is enough to prove it for a1,...,@y > 0 (otherwise we replace 4, 42,-.-+4y with Ja), |az|,..-,|@n|, which have product 1. Yet, the right hand side increases). Now, let a, the maximum number among a),2,...,dx and let G the geometric mean of 1,435. n-1. Fitst, we will prove that Vir Way tay $+ ag abe adtest ab — Dak +(n-1)G* Vir Tay, +(n ~1)G =n) which is equivalent to a} ad+- ab y(n 1h. > (Old and New Ineavalities 105 RAT (ay + ay +++ + ay —(n —1) Yat) - 2n > oni Because, YGF —-tyat < Land ay tazt--t aya (nl) yee 0, it is enough to prove the inequality Qn ate e aly Vna1-G (ay +++ + an-1=(n = 1G), =e a Now, we apply the inductive hypothesis for the numbers 22, which have G product 1 and we infer that aot nti and so it suffices to prove that BOD Bart tay (n— An=1), 1 vin n= 3 = Yan (am x)" "> te and it follows for n> 4 from Lyne) — D2 ( nw 5) ‘This becomes which is the same as 1 + (n—1y ° and (v=1"" mae aaa (Mara) ( For n= Sand n= itis easy to check. ‘Thus, we have proved that VI Tay tarot ay— nj > 2 ae aj+a3 ni 2n Salt (n-G?=n + Yn Tay +(n 1G ~n) and it is enough to prove that a4 Pt ns BE oa (ats i for all >1 (we took r= a Let us consider the function feet -m=I( We have F(x) =2 106 Solutions because 1 1 ol L pot + Grae te? "aot ‘Thus, fis increasing and so f(z) > f(1) = 0. This proves the inequality rly 113. [ Vasile Cirtoaje ] I a,b,c are positive real numbers, then [Ba [Be / <3. Vara ea Gazeta Matematica First solution: With the notations 2 = that xy2=1 implies and we need to prove that Because we only need to prove that ‘This inequality is equivalent to —2/22(1 +2) 20, (Vz - Vz 1) 20, i+ and we are done. (Old and New Ineavalities 107 Second solution: Clearly, the problem asks to prove that if xyz =1 then = Lym We have two cases. The first and easy one is when ty + y2 + 22 >a+y +z. Inthis case we can apply the Cauchy-Schwarz Inequality to get [z= LVmsV 2 om & YL eyte+ © rtyteS VieyeysD which follows from \/2ry(2y +1) < 2/79 b > ¢. If 2c? > ab, each term in the above expression is positive and we are done, So, let 22 < ab, First, we prove that 26? > ae, 2a? > be. Suppose that 26? < ae. Then (b+ 0)? <2(0? +e) 3(zy + ye +21), we get a> 1. Now we write the inequality as follows 1 1 ata (3a — 2)? © (3a—y)? © (3a ~ sr)? Al(zy + 3az)" + (yz + 3ax)? + (20 + 3ay)?] > 3(9a — xyz)", 4(27a" — 18a? +3 + daxye) > (9a ary2)* 3(120? = 1)(3a? — 4) +2y2(34a—2y2)>0, (1) (Old andl New Ineawalities 109 12(3a" = 1)? + 208a? > (17a — yz)’. (2) We have two cases 4) Case 3a? — 40. Since Ha ~ rye = jboate tut 2)(ey + yz + 22) — 9ry2] >0, the inequality (1) is true. ii) Case 3a” — 4. <0, From Schur’s Inequality (ety +29 Ale ty + 2)xy tye + 2x) +9ny2 20, it follows that 3a" — da + 2y2>0. Thus, 12(3a? — 1)? + 2080” — (17a — ayz)* > 12(3a* — 1) + 2080 — a (3a? + 13)? = =3(4— 1a? + 10a — 3a") =3(1 — a?)?(4— 3a? > 0. 115. Prove that for any x,y in the interval [0,1], Vises Vises VA= aS 2 (1+ VIN 2) Solution (by Faruk F. Abi-Khuzam and Roy Barbara - "A sharp in- equality and the iradius conjecture”): Let the function Fo: [0,1P +3 RF(rjy) = ViFe+ fIF¥R+ T= aPC yt = (1+ V5)(1—2y). is clear that Fis symmotii in x and xy and also the convexity of the function x + V+ shows that F(.r,0) > 0 for all :r. Now, suppose we fix y and consider F asa function in «. It’s derivatives are lee = (1+ V8 + Fs Toso and ( 1 = Ware? * asap + aeny ‘Thus, f is convex and its derivative is increasing. Now, let r=, /1- a+ v5? 1 The frst case we will diseuss is y > 5. It is easy to see that in this case we have cy > 1 f(x) a14V5 Ty (the derivative ofthe function y — yte*(y? 29 +2)—1 is positive) = By and so (0) > 0, Because fis increasing, we have f(z) > Oand so f is increasing with f(0)= F(0,y) = Fly,0) 20. Thus, in this case the inequality is proved, no Solutions Due to case 1 and to symmetry, it remains to show that the inequality holds in the cases a € le 5 .y €[0,r] and re fr 4 we Ir 4 Tn the frst caso we deduceimmediately that F(a) 0, we will have F(r,y) = f(z) 2 0 for all points (x,y) with ar€ [a + sv [Or] Now, let us discuss the most important case, when 2 € 4 we ni] Let the points 0(0,0),A(1,0),(.1),C(0, 1), M(1,y), N(e,1). ‘The triangle OMN has perimeter Vives Vise + VOR aE oF and area 2 But it is trivial to show that in any triangle with perimeter P and area S we have the ° Inequality $< Fy. Ths, we find that VFI VIEW VT a a V6V3VT= ay > (1+ ¥5)(1—y) due to the fact that xy > r®. The proofis complete. 116. [ Suranyi ] Prove that for any positive real numbers a1,2,--.40n the fol- lowing inequality holds (n= 1lap taht ----ah)-+narag aq 2 (arbaate bag) bagl4-- bam!) Miklos Schweitzer Competition Solution: Again, we will use induction to prove this inequality, but the proof of the inductive step will be again highly non-trivial, Indeed, suppose the inequality is true for n numbers and let us prove it for n + 1 numbers, Due to the symmetry and homogeneity of the inequality, it is enough to prove it under the conditions ay > ay ny and a + ay +++++ aq, = 1. We have to prove that n oat! + nattl +naye Te +a v Ta (1+ ans) (Se + ein) 20. But from the inductive hypothesis we have (m= I)(a} +a Fe +n) + maya ...an BaP! bay! tee pann! (Old and New Ineavalities a and so nansi ]] 4 2 Ons Ph (n Dans i i Using this last inequality, it remains to prove that a1-Sat) = ans (»S5e-3o0r) + Fan01 (Ite + (nan, — ou!) 20. Now, we will break this inequality into net (Heme = Dab. and * a a (sen -S«) = anst (Se -> «) 20. = : a Let us justify these two inequalities, The fist one is pretty obvious [fait (m= at y= at = T] (ai = ener tanga) + (m Dat = ay} it Dany taney: \ lai anga) + (n— Dah, = any] =O. Now, let us prove the second inequality, It can be written as nal! =Saf Sayer (Se-Yer’). Because n )> a? — 7a?! 2 0 (using Chebyshey’s Inequality) and an 1< i it is enough to prove that Ya -yast (»soer-poer’). 1 but this one follows, because na?*+! + aa > a? for all i. Thus, the induetive step is proved. 117. Prove that for any 2),2,-..,2q > Owith product 1, DY wi-ayt2d8 icigicn 1. A generalization of Turkeviei’s inequality na Solutions Solution: Of course, the inequality can be written in the following form r-w-n()- (E>) i ‘We will prove this inequality by induction. The case n = 2 is trivial. Suppose the inequality is true for n ~1 and let us prove it for n. Let Fe1, 2250-420) and let G'= ~/Farj- Fn, Where we have already chosen 2 Its easy to see that the inequality f(t1,22,..-,tn) (n —1)G, $0 it suffices to prove that min{ry,£2,--.,2n}. + G) is equivalent Doak (Se) a(S ne). ‘We will prove that this inequality holds for all s,... 2%, > 0. Because the inequality is homogeneous, it is enough to prove it when G= 1. In this case, from the induction step we already have ySapen-1s (S*) & = and so it suffices to prove that Vajtn-12 Vem & Vier? 20, mo clearly true, ‘Thus, we have proved that flv...) < fle. plete the inductive step, we will prove that /(1i,,G, nm am , the last assertion reduces to proving that --1-G). Now, to com- :G) <0. Because clearly (n-1) (no? as) tmz (w-ne+gh) which comes down to 2 2n > Gt" and this one is an immediate consequence of the AM-GM Inequality, (Old and New Ineavalities na 118. [ Gabriel Dospinescu ] Find the minim La mn value of the expression T-(m—Nay where @),25.-.54n < 7 add up to 1 and n> 2is an integer Solution: ‘We will prove that the minimal value is Indeed, using Suranyi’s In- equality, wefind that (oY al + raids. .ag 2 ral nasa te Soa" = (na) Now, let us observe that Le Sas in-Da) => and s0, by an immediate application of Hélder Inequality, we have Lap tas (n= ta) > But for n> 3, we can apply Jensen’s Inequality to deduce that Ya ‘Thus, combining all these inequalities, we have proved the inequality for n > 3. For n= 3 it reduces to proving that uy which was proved in the solution of the problem 107, 119. [ Vasile Cirtoaje ] Let a),02,...,dn <1 be nonnegative real numbers stich that 14 Solutions Prove that Solution: We proceed by induetion, Clearly, the inequality is trivial for n= 1. Now we suppose that the inequality is valid for n= — 1, k>2and will prove that for oS We assume, without loss of generality, that ax > a2 > Using the notation From we obtain and ax)(1-+ aay) Ye ~a}(1 + az) ax)(1 + ax) C-@-@) * Gwe) Tw | ma ee ra) Lea? +af + ea ale tas) +02 T= a Vl = al Pye Fa) ka-ax)(a+ax) ~Te-t n(x — a4) wag (z+ a4) + a2 ka’ Fe a= it follows that Ka -ax)*(a+ an) NeW au) (oF ai) 59, (Dera) = (aay) (x ~ ay Old and New Ineaualites nis and hence we have to show that rhb ab + ray + a(t + ay) +a? + aray(e+ ay) +a% ray > 1 In order to show this inequality, we notice that ka? + (k-2)a2 71? 3 E B4a,> for+ap>ay > 40 bray +ale-+ay) +0? +aray(x-+ay) aay > (2 +03) +(e bag) +a > 2(atrvVe) o> and the proof is complete. Equality holds when a, = a2 = Remarks. 1. From the final solution, we can easily see that the inequality is valid for the larger condition Pah and Consequently, we have = aq. ‘This is the largest range for a, because for a1 = a2 = (therefore a: = ay) = and dy = ), from the given inequality we get a > v3 2. The special case n = 3 and a is a problem from Crux 2003. 3 120. [ Vasile Cirtoaje, Mircea Laseu ] Let a,b,c, 2,1, 2 be positive real numbers such that (@tbtoletyt)=@+P+ Arty +2)=4. Prove that 1 berye < abexy2 < 55 Solution: Using the AM-GM Inequality, we have A{ab+be+ea)(xy+yz+zr) = [(a +b +0)? = (a? +0? +e°)] [ety +2) -(? +97 +22)] = =W-(atbteP (a? +y+2)— (PLP Feller y tes $20-2V at bt PET +L PT Ae ty tay =4, a6 Solutions therefore (ab+be+ca)(ry+y2+2a)<1. (1) By multiplying the well-known inequalities (ab + be + ca)? > Babe(a+b+e), (xy +yz+t2x)* 2 Scy2(xt+y +2), it follows that (ab + be + ea)°(xy + yz + 22)? > Yabery2(a+b+e)(x+y +z), or (ab + be + ca)(ey + ye + 22) > 36abery2. (2) From (1) and (2), we conclude that 1< (ab+ be+ ca) (ey + y2 + 20) > 3abery2, therefore 1 < 36abezy2. To have 1 = 36abezyz, the equalities (ab +be+ ca)? = Sabe(a + b+e) and (y+ y2-+ 22)? = 3rye(x+y +2) are necessary. But these conditions imply a= b = and x = y= 2, which contradict the relations (a + + ¢)(e-+ y +2) = (a? 40+ 2)(a? + y+ 2) = 4. Thus, it follows that 1 > 36abery2. 121, [ Gabriel Dospinescu ] For a given n > 2, find the minimal value of the constant fy, such that if 2 23,...yz»> O have product 1, then 1 1 1 Vein Wein Fee S Mattlinks Contest Solution: We wil prove that by = 2™ 55. Takings = 2 =1, we find that ky 2 PEA. So, it remains to show that @-1y 1 0. Thus, we can find a number M>n=1 and some nutbers a; > O which add up to 1, such that = Mag, (Old andl New Ineawalities ut I> Ge) (2n=1)" (ite - 5) i a Because from the AM-GM Inequality we have Te-w<(#) . our assumption leads to Thr by? < tate So, it is enough to prove that for any positive numbers a), a2, -..,4y the inequality i (n — 1)?" TD lant aa te pan a bang te ban)? > oo ms holds. ‘This strong inequality will be proved by induction. For n= fact that 2 (8 2 (eH), E99) ougyay abe abe! “27 % ‘Suppose the inequality is true for all systems of n numbers. Let a1,a2,...,@n+1 be positive real numbers. Because the inequality is symmetric and homogeneous, we may assume that @ < a2 <+++ eo, ‘To prove the inductive step, we must prove that 01... dn (ay +02 +°-++ an)” , it follows from the 110 My cecaet-ay> 22 It inet)? > ya ‘Thus, it is enough to prove the stronger inequality II (: + 050). a(1 + Ong)" int? a Tae y paring (l dost us Solutions Now, using Huygens Inequality and the AM-GM Inequality, we find that 2 (1-3) ‘ fn) = and so we are left with the inequality wena" pine? i (=) > ral tae) ) nl ta 1 if anes 2 max{a,,aa,...,4n} 2 =. So, weean pu 14 x, where wis nonnegative. So, the inequality becomes an “Abt de 1 Li{nt Ye ( +aesn) = erp Using Bernoulli Inequality, we find immediately that Also, (1 +2)! 51.4 (n 1a and so it is enough to prove that Betis 1e(nt te Fal *Ts@—Ne nel which is trivial. So, we have reached a contradiction assuming the inequality doesn’t hold for a certain system of n numbers with product 1, which shows that in fact the inequality is true for ky = 2 7 and that this is the value asked by the problem. ‘Remark. Forn }, we find an inequality stronger than a problem given in China Math- ematical Olympiad in 2003. Also, the case n = 3 represents a problem proposed by Vasile Cartoaje in Gazeta Matematici, Seria A 122. [ Vasile Cirtoaje, Gabriel Dospinescu ] For a given n > 2, find the maximal value of the constant ky such that for any 21,2,.+.,2_ >0 for which xf +03 4-6-4 22 = L we have the inequality (1=2y)(1— 22) (Lt) > ara (Old andl New Ineawalities no Solution: ‘We will prove that this constant is (yi —1)". Indeed, let a; = the minimal value of the expression 2, We must find Tla-va) f1_ ve it when a; +a2+-+--+a, = 1. Let us observe that proving that this minimum is (/—1)" reduces to proving that TI -ad 2 (va-1y" = But from the result proved in the solution of the problem 121, we find that ee Tla- a's (*) The. So, it is enough to prove that [larva (1+) But this is an easy task, because from the AM-GM Inequality we get <() the last one being a simple consequence of the Cauchy-Schwarz Inequality. Remark. ‘The case n = 4 was proposed by Vasile Cartoaje in Gazeta Matematica Annual Contest, 2001 Glossary (1) Abel's Summation Formula Wh 4154-054, Bisby, ns by ave real or complex numbers, and Lap teeet agi = 12,57, then 41) + Sudo. Yan=Ts0.- (2) AM-GM (Arithmetic Mean-Geometric Mean) Inequality Tf @,03,...,dy are nonnegative real numbers, then Yai > (ayay...04)* with equality if and only if a1 = a2 case of the Power Mean Inequality. L ay. T is inequality is a special (3) Arithmetic Mean-Harmonic Mean (AM-HM) Inequality Tf ay,42, --4q are positive real numbers, then with equality if and only if a case of the Power Mean Inequality. aq. This inequality is a special (4) Bernoulli’s Inequality For any real numbers > —Land a> 1 we have (1+2)" > 1+ ar, a Solutions (5) Cauchy-Schwarz’s Inequality For any real numbers ay,a3,..,y a0 By bay ae (a taj tu. taht b+ + BS > (aibi + aaby +... + anda)? with equality if and only if a; and by are proportional, §= 1,2,...1. (6) Cauchy-Schwarz’s Inequality for integrals Tf.a,b are real mmbers, a = (Se): (4): 2 by then S7 aibi< t (Se): (ds): (8) Chebyshev’s Inequality for integrals Ia, bare real numbers, a [ * plalde [ * pene and if one is increasing, while the other is decreasing the reversed inequality is true. 2)IF by > be > (9) Convex funetion A real-valued function f defined on an interval 7 of real numbers is convex if, for any x,y in J and any nonnegative numbers a, with sum 1, Slow + By) Saf(x) + 8f(y) (10) Convexity A function f(x) is concave up (down) on [a,b] R if f(x) lies under (over) the line connecting (a;, f(a1)) and (bi, f(b1)) for all a [or = TT 1 (14) Mae Laurin’s Inequality For any positive real numbers 21,2... S12 S.> Solutions where (15) Minkowski's Inequality For any real number r > 1 and any positive real numbers 0,49, .6 +5 ny Bi, Ba, Dny 3 (Er) Bs)“ (16) Power Mean Inequality For any positive real numbers a;,02,...,dy with sum equal to 1, and any positive real munbers #1,72,...,m, We define My = (ay + ag} + set ant)? if is a non-zero real and My = af'r3*...2%*, Mio = macr{y,22,+.-5tn}; M-oo = min{21,£2,...4Z,}. Then for any reals s <¢ we have Moco < My 0, 2"(r—y)e—2)+y"(y— 2)(y—2) + 2"(—2)(z—y) 20. The most common case ist = 1, which has the following equivalent forms: 1) ae yb e84 Saye Dave ty) + vely +2)+ eels +2) 2) ayz>(tt+y—2)yt+z—2\z+2-y); 3) if ety tes then nytysesr< Lier (19) Suranyi’s Inequality For any nonnegative real numbers ay ,02,-.-,4n, (on Sat nT (S4)-(Se"). Old and New Ineaualites 125 (20) Turkeviei’s Inequality For any positive real numbers x, y,2,¢, city tet titre y sys PPT Pe eet ay E (21) Weighted AM-GM Inequality For any nonnegative real mambers a), 25.10, n, ft, 2, 00 ty are nonneg- ative real numbers (weights) with sum 1, then way + wa +. Wty 2 ay" ay? a", with equality if and only if a = a: Oy. Further reading 1. Andreescu, T.; Feng, Z., 101 Problems in Algebra from the Training of the USA IMO Team, Australian Mathematics Trust, 2001 2. Andreescu, T.; Feng, Z., USA and International Mathematical Olympiads 2000, 2001, 2002, 2003, MAA. 3. Andreescu, T.; Feng, Z., Mathematical Olympiads: Problems and Solutions from around the World, 1998-1999, 1999-2000, MAA. 4, Andreescu, T.; Kedlaya, K., Mathematical Contests 1995-1996, 1996-1991, 1991-1998: Olympiad Problems from around the World, with Solutions, AMC. 5. Andreescu, T.; Enescu, B., Mathernatical Olympiad Treasures, Birkhdiuser, 2003. 6. Andreescu, T.; Gelea, R., Mathematical Olympiad Challenges, Birkhduser, 2000. 7. Andreescu, T.; Andrica, D., 260 Problems for Mathematical Contests, GIL Publishing House, 2002 8. Becheanu, M., Enescu, B., Inegalitafi elementare, .. si mai putin elementare, GIL Publishing House, 2002 9. Beckenbach, E., Bellman, R. Inequalities, Springer Verlag, Berlin, 1961 10. Drimbe, M.O., Inegalitifi idei si metode, GIT, Publishing House, 2003, 11. Engel, A., Problem-Solving Strategies, Problera Books in Mathematics, Springer, 1998. 12. G.H. Littlewood, J.B, Polya, G., Inequalities, Cambridge University Press, 1967 13. Klamkin, M., International Mathematical Olympiads, 1978-1985, New Mathematical Library, Vol, $1, MAA, 1986 14. Larson, L..C., Problem-Solving Through Problems, Springer-Verlag, 1983. 15, Lascu, M., Inegalititi, GIL Publishing House, 1994 16, Liu, A., Hungarian Problem Book ITT, New Mathematical Library, Vol. 42, MAA, 2001 17, Lozansky, B; Rousseau, C., Winning Solutions, Springer, 1996 18, Mitrinovie, D.S., Analytic inequalities, Springer Verlag, 1970 19, Panaitopol, L., Biindili, V., Lascu, M., Inegalitéti, GIL Publishing House, 1995 20, Savchev, S.; Andreescu, T., Mathematical Miniatures, Anneli Lax New Mathematical Library, Vol. 43, MAA. ‘wow amathlinks.r0~ Powered by www gil.ra at

You might also like